Вы находитесь на странице: 1из 106

Item: 1 of 16 ~ 1 • M k -<:J 1>- Jil ~· !

:';-~
QIO: 4571 ..L ar Pre v ious Next Lab fli!ltues Not es Calcula t o r

•1

.2
A 53-year-old woman with years of poorly contro lled type 2 diabetes mellitus developed rena l failure and was IAA] •

placed hemodialysis while awaiting a transplant. She eventually had a renal transplant and completed the
•3 induction immunosuppressant regimen w ithout experiencing any signs of rejection . She presents today to
·4 start the maintenance course of her immunosuppressant regimen, in which one of the main com ponents is
•5
tacrolimus (FK506), a common ly used immunosuppressant agent in transplant surgery.
•6
What is the mechanism of action of tacrolimus?
.7
·8 :
A . Complex with cyclophilin
.9
• 10 B. Directly binding the interleukin-2 receptor
• 11 C. Inhibition of calcineurin
• 12
D. I nhib ition of dihydrofolate red uctase
• 13
• 14
E. I nhib ition of DNA synthesis
• 15
• 16

a
Lock
s
Suspend
8
End Bl ock
Item: 1 of 16 ~. , . M k <:] t> al ~· ~
QIO: 4571 .l. ar Previous Next lab 'lifllues Notes Calculator

1
. 2 The correct answer is C. 57°/o chose this.
•3 Tacrolimus acts by complexing with FK-bind ing protein 12, an immunoph ilin, and then inh ibiting calcineurin.
.4 This leads to the downstream effect of inhibiting T-lymphocyte signal ing. Tacro limus is metabolized by
cytochrome P-450 enzyme 3A.
•5
FKBP Tacrolimus Immunophilins Calcineurin Enzyme T cell Cytochrome P450 Protein Metabolism Cytochrome
•6
.7
A is not correct. 9°/o chose this.
Cyclospor in, like tacrolimus, also functions by inh ibiting calcineurin. The key diffe rence, however, is that
•8
cyclosporin complexes with cyclophil in, whereas tacrol imus complexes with FK-binding protein 12 .
•9 FKBP Tacrolimus Calcineurin Ciclosporin Cyclophilin Protein

• 10
B is not correct. 22% chose this.
· 11
Monoclonal antibody therapies, such as dacl izumab, di rectly bind to the interleukin-2 (I L-2) recepto r and
• 12 prevent clonal expansion ofT lymphocytes. Although tacro limus also affects the action of IL-2, it does not
• 13 directly bind the I L-2 receptor .
Daclizumab Tacrolimus Interleukin 2 Monoclonal antibody Il-2 receptor Antibody lymphocyte T cell Cloning Receptor (biochemistry)
• 14
• 15 D is not correct. 6°/o chose this .
• 16 Methotrexate is both an immunosuppressive agent and a chemotherapeutic agent . By blocking dihydrofo late
reductase, it leads to inhibition of DNA synthesis.
Methotrexate Dihydrofolate reductase Immunosuppressive drug Immunosuppression Chemotherapy DNA replication DNA Dihydrofolic acid

E is not correct. 6°/o chose this.


Azathiopr ine is an immunosuppressive agent that inhibits DNA synthesis by acting like a purine analog, and
mycophenolate inh ibits pu r ine synthesis by inhibiting inosine monophosphate dehydrogenase (IMPDH) . They are
used in the treatment of autoimmune diseases and the prevention of orqan transplant reiection. •
6
lock
s
Suspend
0
End Block
Item: 1 of 16 ~. , . M k <:] t> al ~· ~
QIO: 4571 .l. ar Previous Next lab 'lifllues Notes Calculator

1 cyclosporin complexes with cyclophi lin, wh ereas tacrolimus complexes with FK-binding protein 12 .
FKBP Tacrolimus Calcineurin Ciclosporin Cyclophilin Protein
. 2
•3 B is not correct. 22% chose this.
.4 Monoclonal antibody t herapies, such as daclizumab, di rectly bind to t he int erleukin- 2 (I L-2) receptor and
•5
prevent clonal expansion of T lymphocytes. Although t acrolimus also affects t he action of IL- 2, it does not
directly bind t he I L-2 receptor .
•6 Daclizumab Tacrolimus Interleukin 2 Monoclonal antibody Il-2 receptor Antibody lymphocyte T cell Cloning Receptor (biochemistry)
.7
D is not correct. 6°/o c hose this .
•8
Methotrexate is both an immunosuppressive agent and a chemotherapeutic agent . By blocking dihyd rofo late
•9
reductase, it leads to inhibit ion of DNA synthesis .
• 10 Methotrexate Dihydrofolate reductase Immunosuppressive drug Immunosuppression Chemotherapy DNA replication DNA Dihydrofolic acid

· 11
E is not correct. 6°/o c hose this .
• 12 Azat hiopr ine is an immunosuppressive agent t hat inhibits DNA synt hesis by acting like a purine ana log, and
• 13 mycophenolat e inh ibits pu r ine synt hesis by inhibit ing inosine monophosphate dehyd rogenase (IMPDH) . They are
• 14 used in t he t reatment of autoimmune diseases and t he prevention of organ transplant rejection .
Azathioprine IMP dehydrogenase Purine Inosinic acid Purine analogue Organ transplantation Transplant rejection Immunosuppressive drug Inosine
• 15
Autoimmune disease Immunosuppression Mycophenolic acid Autoimmunity DNA DNA replication
• 16

Bottom Line:
Tacrolimus inhibits ca lcineu rin by comp lexing with FK-binding protein 12 .
FKBP Tacrolimus Calcineurin Protein

6
lock
s
Suspend
0
End Block
Item: 1 of 16 ~. , . M k <:] t> al ~· ~
QIO: 4571 .l. ar Previous Next lab 'lifllues Notes Calculator

1 •

•2 FA17 p 116.1
•3 Immunosuppressants Agents that block lymphocyte activation and proliferation. Reduce acute transplant rejection by
•4 suppressing cellular immunity (used as prophylaxis). Frequently combined to achieve greater
•5 efficacy with ! toxicity. Chronic suppression t risk of infection and malignancy.
DRUG MECHANISM OTHERUSE TOXICITY NOTES
•6
.7
Cyclosporine Calcineurin inhibitor; Psoriasis, rheumatoid Nephrotoxicity,
binds cyclophilin. arthritis. hypertension,
•8 Blocks T-cell hyper!ipidemia,
•9 activation by neurotoxicity,
• 10 preventing IL-2 gingival hyperplasia,
transcription. hirsutism. Both calcincurin
· 11 inhibitors are highly
• 12
Tacrolimus (FK506) Calcineurin inhibitor; Similar to cyclosporine, nephrotoxic .
binds FK506 binding t risk of diabetes
• 13
protein (FKBP). and neurotoxicity;
• 14 Blocks T-cell activation no gingival
• 15 by preventing IL-2 hyperplasia or
• 16 transcription . hirsutism.
Sirolimus (Rapamycin) mTOR inhibitor; binds "PanSirtopenia" Kidney "sir-vives."
FKBP. (pancytopenia), Synergistic with
Blocks T-cell insulin resistance, cyclosporine.
activation and B-cell Kidney transplant hyperlipidemia; Also used in cl rug-
differentiation by rejection prophylaxis not nephrotoxic. eluting stents.
preventing response specifically.
TT .., •

6
lock
s
Suspend
0
End Block
Item: 1 of 16 ~ 1 • M k -<:J 1>- Jil ~· !:';-~
QIO: 4571 ..L ar Pre v ious Next Lab fli!ltues Not es Calcula t o r

1
Sirolimus (Rapamycin) mTOR inhibitor; binds "PanSirtopen ia" Kidney "sir-\ i' es."
•2
FKBP. (pancytopenia), Synergistic with
•3
Blocks T-cell insulin resistance, cyclospori ne.
·4 activation and B-cell Kidney transplant h) perlipidemia; Also used in drug-
•5 differentiation by rejection prophylaxis not ncplu otoxic. eluting stents.
•6
preventing response spccilically.
to IL-2.
.7
Basiliximab ~ lonoclonalantibody; Edema, hypertension,
·8
blocks IL-2R. tremor.
.9
Azathioprine Antimetabolite Rheumatoid arthritis, Pane} topenia. 6-YIP degraded by
• 10 precursor of Crohn disease, xanthine oxidase;
• 11 6-mercaptopurine. glomerulonephritis, toxicity t by
• 12 Inhibits lymphocyte other autoimmune allopurinol.
proliferation by cond itions. Pronounce '·azathio-
• 13 . ."
blocking nucleotide purme
• 14
synthesis .
• 15
Mycophenolate Reversibly inhibits Lupus nephritis. Gl upset, Associated with
• 16 mofetil IMP dehydrogenase, pancytopenia, invasive CMV
preventing purine hypertension, infection.
synthesis of B and T hyperglycemia.
cells. Less nephrotoxic and
neurotoxic.
Corticosteroids Inhibit lF-KB. 'vlan\'• autoimmune Cushing syndrome, Demargination
Suooress both B- and and inAammatorv osteooorosis. of\VBCs causes
a
Lock
s
Suspend
8
End Bl ock
Item: 1 of 16 ~ 1 • M k -<:J 1>- Jil ~· !:';-~
QIO: 4571 ..L ar Pre v ious Next Lab fli!ltues Not es Calcula t o r

1 •
blocks IL-2R. tremor.
•2
Azathioprine Antimetabolite Rheumatoid arthritis, Pancytopenia. 6-M P degraded by
•3 precursor of Crohn disease, xanthine oxidase;
·4 6-mercaptopurine. glomerulonephritis, toxicity t by
•5
Inhibits lymphocyte other autoimmune allopurinol.
proliferation b~ conditions. Pronounce '·azathio-
•6
blocking nucleotide . "
purmc.
.7 srnthesis.
·8 Mycophenolate Re,·ersiblv• inhibits Lupus nephritis. Gl upset. Associated with
.9 mofetil I IP dehydrogenase, pancytopenia, im·asive C~!fV
• 10 preventing purine b) pertension, infection.
• 11
synthesis of B and T hrperglycemia.
cells. Less nephrotoxic and
• 12
neurotoxic .
• 13
Corticosteroids Inhibit 1 F-KB. Many autoimmune Cushing syndrome, Demargination
• 14 Suppress both B-and and inAammatory osteoporosis, ofWBCs causes
• 15 T-ccll function by disorders, adrenal hyperglycemia, artificial leukocytosis.
• 16 l transcription of insufficiency, asthma, diabetes, amenorrhea, Adrenal insufficiency
many cytokines. CLL, non-Hodgkin adrenocortical may develop if drug is
Induce T cell apoptosis. lymphoma. atrophy, peptic ulcers, stopped abruptly after
psychosis, cataracts, chronic use.
m·ascular necrosis
(femoral head).

a
Lock
s
Suspend
8
End Bl ock
Item: 2 of 16 ~ 1 • M k -<J t:>- •I ~· Ill
QIO: 3657 ..L ar Previous Next Lab Wues N'otes Calculator

1
A 42-year-old woman presents to her doctor's office complaining of worsen ing fatigue . She has also recently ~~
.2
noticed occasional difficulty in open ing her left eye. She states that she feels f ine when she wakes up, but
•3 her head " feels heavy" towards the end of the day.
.4
•5 Which medication is used for treatment for this condition?
•6
:
.7 A. Atropine
·8 B. Edrophonium Error
.9
C. Pilocarpine Unable to answer question. Please try again .
• 10
• 11 D. Pyridostigmine
• 12 E. Vecuronium
• 13
• 14
• 15
• 16

a
Lock
s
Suspend
o
End Block
Item: 2 of 16 ~ 1 • M k -<:J 1>- Jil ~· !:';-~
QIO: 3657 ..L ar Pre v ious Next Labfli!llues Not es Calcula t o r

1
.2
A 42-year-old woman presents t o her doctor's office complaining of worsen ing fatigu e. She has also recently IAA] •

noticed occasional difficulty in opening her left eye. She st ates that she feels f ine when she wakes up, but
•3 her head "feels heavy" towards the end of the day.
·4
•5 Which medication is used for treatment for this condition?
•6
:
.7 A . Atropine
·8 B. Edrophonium
.9
C. Pilocarpine
• 10
• 11 D. Pyridostigmine
• 12 E. Vecu ron ium
• 13
• 14
• 15
• 16

a
Lock
s
Suspend
8
End Bl ock
Item: 2 of 16 ~. , . M k <:] t> al ~· ~
QIO: 3657 .l. ar Previous Next lab 'lifllues Notes Calculator

1 The correct answer is D. 71°/o chose this.


2 This patient has myasthenia g ravis. Pyridostigmine is an acetylchol inesterase inhibitor used in the management
•3 of chronic myasthenia gravis and has du ration of 3-6 hours. Pyridostigmine inh ibits acetylcholinesterase in the
.4
synaptic cleft by competing with acetylcholine for attachment to acetylchol inesterase, thus prolonging the
effects of acetylchol ine. Neostigmine is also a acetylchol inesterase inhibitor but its du ration of action is on ly 2-4
•5 hou rs. Thymectomy is f irst- line treatment in patients with genera lized myasthenia gravis.
•6 Acetylcholinesterase inhibitor Myasthenia gravis Neostigmine Synaptic cleft Pyridostigmine Acetylcholinesterase Acetylcholine Chemical synapse Thymectomy

.7 Enzyme inhibitor

•8
A is not correct. 6°/o chose this .
•9 Atropine is an antichol inergic agent. It has a high affi nity for muscarinic receptors and is both a central and a
• 10 peripheral muscarinic receptor blocker. It is not used for treatment of myasthenia gravis .
Anticholinergic Myasthenia gravis Atropine Muscarinic acetylcholine receptor Receptor (biochemistry)
· 11
• 12 B is not correct. 13% chose this .
• 13 Edrophonium is a read ily reversible acetylchol inesterase inhibitor used in the Tensilon test to he lp diagnose
• 14 myasthenia gravis. However, its duration of action is on ly 19 minutes and it is thus impractical as a t reatment
for the disease .
• 15
Acetylcholinesterase inhibitor Myasthenia gravis Acetylcholinesterase Edrophonium Myasthenia Enzyme inhibitor
• 16
C is not correct. 8°/o chose this.
Pilocar pine stimu lates muscarinic cho linergic receptors and is the drug of choice in the emergency lowering of
intraocu lar pressure of both closed-ang le and open -angle glaucoma . It does not have a role in the treatment of
myasthenia gravis.
Pilocarpine Myasthenia gravis Glaucoma Intraocular pressure Cholinergic Muscarinic acetylcholine receptor Receptor (biochemistry)

E is not correct. 2°/o chose this.


6
lock
s
Suspend
0
End Block
Item: 2 of 16 ~. , . M k <:] t> al ~· ~
QIO: 3657

1
.l.
.. p
ar
g
Previous
g
Next

g y
lab 'lifllues Notes
p
Calculator

peripheral muscarinic receptor blocker. It is not used for treatment of myasthenia gravis.
.. .
2
Anticholinergic Myasthenia gravis Atropine Muscarinic acetylcholine receptor Receptor (biochemistry)
•3
.4
B is not correct. 13% chose this.
Edrophonium is a read ily reversible acetylcholinesterase inhibitor used in the Tensilon test to help diagnose
•5
myasthenia gravis. However, its duration of action is on ly 19 minutes and it is thus impractical as a t reatment
•6 for the disease.
.7 Acetylcholinesterase inhibitor Myasthenia gravis Acetylcholinesterase Edrophonium Myasthenia Enzyme inhibitor

•8
C is not correct. 8°/o chose this .
•9 Pilocar pine stimu lates muscarinic cholinergic receptors and is the drug of choice in the emergency lowering of
• 10 intraocu lar pressure of both closed-angle and open-angle glaucoma . I t does not have a role in the treatment of
· 11
myasthenia gravis.
Pilocarpine Myasthenia gravis Glaucoma Intraocular pressure Cholinergic Muscarinic acetylcholine receptor Receptor (biochemistry)
• 12
• 13 E is not correct. 2°/o chose this .
• 14 Vecuronium is a nonpolarizing neu romuscular antagonist. It is used as a pa ralytic agent du r ing surgery and is
not used in the t reatment of myasthen ia gravis .
• 15
Myasthenia gravis Vecuronium bromide Neuromuscular-blocking drug Neuromuscular junction Receptor antagonist
• 16

Bottom Line:
Pyridostigmine is an acetylchol inesterase inhibitor used in the management of ch ron ic myasthenia gravis.
Acetylcholinesterase inhibitor Myasthenia gravis Pyridostigmine Acetylcholinesterase Enzyme inhibitor Myasthenia

6
lock
s
Suspend
0
End Block
Item: 2 of 16 ~. , . M k <:] t> al ~· ~
QIO: 3657 .l. ar Previous Next lab 'lifllues Notes Calculator

1
FA17 p 232.1
2
Cholinomimetic agents
•3
DRUG ACTION APPLICATIONS
.4
Direct agonists
•5
Bethanechol Activates bowel and bladder smooth muscle; Postoperative ile us, neurogenic ileus, urinary
•6
resistant to AChE, no nicotinic activity. retention
.7 "Bethany, call (bethanechol) me to acti\·ate
•8 your bowels and blaclclcr."
•9 Carbachol C arbon copy of acetylcholine (but resistant to Constricts pupil and relieves intraocular
• 10 AChE). pressure in open-angle glaucoma
· 11 Methacholine Stimulates m uscarinic receptors in ai rway when Challenge test for diagnosis of asthma
• 12 inhaled .

• 13 Pilocarpine Contracts ciliary muscle of eye (open-angle Potent stimulator of sweat, tears, and saliva
glaucoma), pupillary sphincter (closed-angle Open-angle and closed-angle glaucoma,
• 14
gla ucoma); resistant to AChE, can cross blood- xerostomia (Sjogren syndrome)
• 15
brain barrier (tertiary amine). "You cry, drool,
• 16 and sweat on you r ' pilow."'
Indirect agonists (anticholinesterases)
Galantamine, t ACh. Alzheimer disease (Alzh eimer patients gallantly
donepezil, swim down the river)
rivastigmine
Edrophonium t ACh. Historically used to diagnose myas thenia gravis;

6
lock
s
Suspend
0
End Block
Item: 2 of 16 ~ M k -<:J 1>- Jil ~· !:';-~

.
1 •

QIO: 3657

1
..L ar Pre v ious
...
Next

. ,
Labfli!llues Not es Calcula t o r

glaucoma); resistant to AChE, can cross blood-


.. . -
.. .. . . ~.
.· -·
-
xerostomia (Sjogren syndrome)
.
2
brain barrier (tertiary amine). "You cry, drool,
•3 and sweat on your 'pilow.'"
·4 Indirect agontsts (anticholinesterases)
•5
Galantamine, t ACh . AL,hcimcr disease (AL~:heim cr patients gallantly
•6 donepezil, S\\ im do'' n the ri' er)
.7 rivastigmine
·8 Edrophonium t Ch. Historically used to diagnose myasthenia gravis;
.9 replaced by anti-AChR Ab (anti-acetylcholine
• 10
receptor antibody) test.
• 11
Neostigmine t ACh. Postoperative and neurogenic ileus and
Neo C 1S = No C S penetration (quaternary urinary retention, myasthenia gravis,
• 12
amine). reversal of neuromuscular junction blockade
• 13
(postoperative).
• 14
Physostigmine t ACh. Ph) SOstigm inc "ph) xes" atropine Antidote for anticholinergic toxicity; phrecly
• 15 overdose. (freely) crosses blood-brain barrier _... C IS
• 16 (tertiary amine).
Pyridostigmine t ACh; t muscle strength. Pyridostigm ine gets M)'aslhenia gravis (long acting); does not
rid of myasthenia gravis. penetrate C 1S (quaternary amine).
1 ole: With all cholinomimetic agents, watch for exacerbation ofCOPD, asthma, and peptic ulcers when giving to susceptible
patients.

a
Lock
s
Suspend
8
End Bl ock
Item: 3 of 16 ~ 1 • M k -<:J 1>- Jil ~· !:';-~
QIO: 4573 ..L ar Pre v ious Next Lab fli!ltues Notes Calcula t o r

1 •
A 40-year-old man with a long history of medical ly m anaged Crohn disease presents w ith shortness of
2
breath . He complains of repeated incidents of bruising following minor t rauma and increasing weakness.
•3 Laboratory tests show:
·4
Hemoglobin : 5.5 g/ dl
•5
Hematocrit: 13%
•6 WBC count: 2200/ mm 3
.7 Platelet count: 39,000/ mm 3
·8 Results from a bone marrow biopsy are shown in the image. A detailed history reveals that the patient had an
.9 acute episode of gout 3 months ago and was prescribed a medicine to prevent future attacks .
• 10
• 11

• 12
• 13
• 14
• 15
• 16

a
Lock
s
Suspend
8
End Bl ock
Item: 3 of 16 ~ 1 • M k -<:J 1>- Jil ~· !:';-~
QIO: 4573 ..L ar Pre v ious Next Lab fli!ltues Notes Calcula t o r
-- . I I I I I I
1
Hematocrit: 13%
2
WBC count: 2200/mm3
•3 Platelet count: 39,000/mm 3
·4 Results from a bone marrow biopsy are shown in the image. A detai led history reveals that the patient had an
•5 acute episode of gout 3 months ago and was prescribed a medicine to prevent future attacks.
•6
.7
·8
.9
• 10
• 11

• 12
• 13
• 14
• 15
• 16

Adverse interactions of which two drugs most likely caused this patient's condition?

a
Lock
s
Suspend
8
End Bl ock
2
•3

·4
•5
•6
.7
·8
.9
• 10

• 11

• 12
Adverse interactions of wh ich two drugs most li kely caused th is patient's condition?
• 13
:
• 14
A. Azathioprine and allopurinol
• 15
• 16
B. Azathioprine and colchicine
C. Infliximab and colch icine
D . Predn isone and allopurinol
E. Predn isone and colchicine

a
Lock
s
Suspend
8
End Block
Item: 3 of 16 ~. , . M k <:] t> al ~· ~
QIO: 4573 .l. ar Previous Next lab 'lifllues Notes Calculator

2
The correct answer is A. 50°/o chose this.
3 Azathiopr ine is a common treatment for Crohn disease; its metabo lite, 6-mercaptopu rine, is normally
.4 inactivated by xanthine oxidase. Allopu r inol and febuxostat are xanthine oxidase inh ibitors used to prevent
•5 repeat gout attacks. When azathiop rine is used concomitantly with either of these medications, they cou ld
•6
interfere with the metabolism of azath ioprine's toxic metabolite, which can lead to fata l hematologic diso rders,
such as aplastic anemia . Th is diagnosis is confi rmed in the photomicrograph of the bone marrow biopsy, which
.7 shows aplastic anemia: marked ly hypocel lular bone marrow (predominantly fat), with rare hematopoetic
•8 precurso r cel ls, but no atypical cells .
Azathioprine Xanthine oxidase Allopurinol Mercaptopurine Xanthine Gout Crohn' s disease Aplastic anemia Bone marrow examination Biopsy Metabolite Anemia
•9
Bone marrow Metabolism Hematology
• 10

· 11 B is not correct. 16% chose this.


• 12 Colch icine can be used to t reat acute gout or provide prophylaxis against futu re gout attacks . It binds to tubu lin
• 13 and prevents microtubu le polymerization. Rarely, colchicine has been reported to cause ap lastic
anemia/pancytopen ia; however, it is in no way related to azath ioprine metabolism . Therefo re, colchicine and
• 14
azathiop rine cou ld be used simultaneously in the setting of an acute gout f lare with no increased r isk of aplastic
• 15 anemia/pancytopen ia compa red with use of colchicine alone .
• 16 Microtubule Colchicine Azathioprine Gout Tubulin Metabolism Polymerization

C is not correct. 10% chose this.


I nfl iximab is an antitumor necrosis facto r-a monoclona l antibody sometimes used to treat Crohn disease . It is
not known to interact with colchicine to cause pancytopen ia.
Infliximab Pancytopenia Colchicine Crohn' s disease Monoclonal antibody Antibody Necrosis

D is not correct. 14% chose this.

6
lock
s
Suspend
0
End Block
Item: 3 of 16 ~. , . M k <:] t> al ~· ~
QIO: 4573 .l. ar Previous Next lab 'lifllues Notes Calculator

1 anemia/pancytopen ia; however, it is in no way related to azath ioprine metabolism . Therefore, colchicine and
2 azathioprine cou ld be used simultaneously in the setting of an acute gout flare with no increased r isk of aplastic
anemia/pancytopen ia compared with use of colchicine alone .
3
Microtubule Colchicine Azathioprine Gout Tubulin Metabolism Polymerization
.4
•5
C is not correct. 10% chose this.
•6
I nfl iximab is an antitumor necrosis factor-a monoclona l antibody sometimes used to treat Crohn disease . It is
not known to interact with colchicine to cause pancytopen ia.
.7 Infliximab Pancytopenia Colchicine Crohn's disease Monoclonal antibody Antibody Necrosis
•8
D is not correct. 14% chose this .
•9
Predn isone is a corticosteroid that can be used to manage Crohn disease. It is not known to have fata lly adverse
• 10 interactions such as pancytopenia when used with allopurinol.
· 11 Corticosteroid Allopurinol Prednisone Pancytopenia Crohn's disease

• 12 E is not correct. 10% chose this .


• 13 Although predn isone is a treatment for Crohn disease and colchicine is a treatment for gout, the two can be
• 14 safely combined in treating primary sclerosing cho langitis .
Primary sclerosing cholangitis Prednisone Crohn's disease Colchicine Gout
• 15
• 16

Bottom Line:
When used concomitantly, al lopurinol (and febuxostat) interferes with the metabo lism of azath ioprine's toxic
metabolite, 6-mercaptopu r ine, wh ich can lead to fatal hematologic disorders, such as aplastic anemia.
Azathioprine Allopurinol Mercaptopurine Aplastic anemia Anemia Metabolite Metabolism Febuxostat Hematology

6
lock
s
Suspend
0
End Block
Item: 3 of 16 ~ 1 • M k -<:J 1>- Jil ~· !:';-~
QIO: 4573 ..L ar Pre v ious Next Lab fli!ltues Notes Calcula t o r

1 •
FA17 p 4 17.1
2
Anti met abolites
3
DRUG MECHANISM' CliNICAL USE ADVERSE EFFECTS
·4 Purine (thiol) analogs Prc\'cnling organ rejection,
Azathioprine, .\lrelosuppression, Gl, li\·er.
•5 6-mercaptopurine - l de no\·o purine synthesis. rheumatoid arthritis, IBD. Azathioprine and 6-.\IIP are
•6 Activated bv 1-IGPRT SLE; used to \\Can patients metabolized by xanthine
.7 zathioprine is metabolized off steroids in chronic disease oxidase; thus both ha\'C
into 6-.MP. and to treat steroid-refractory t toxicity with allopurinol or
·8
chronic disease. febuxostat.
.9
Cladribine Purine analog - multiple l lairy cell leukemia. ~ lyelosuppression,
• 10
mechanisms (eg, inhibition nephrotoxicity, and
• 11
of D A polymerase, Dr\ A neurotoxicity.
• 12 strand breaks).
• 13 Cytarabine Pyrimidine am1log- inhibition Leukemi;~s (A!vl L). I} mphomas. ~ lyelosuppression
with
• 14 (arabinofuranosyl of DNA polymerase. megaloblastic anemia.
• 15 cytidine) GYTarabinc causes
• 16
panCYTopen ia .
5-fluorouracil Pyrimidine analog bioactivated Colon cancer, pancreatic Myelosuppression, pa lmar-
to 5-FdUMP, which cancer, basal cell carcinoma plantar eryth rodyseslhe~ia
covalently complexes with (topical). (hand-fool syndrome).
thymidylate synthase and l•:ffecls enhanced with the
foli c acid. Capecitabinc is a addition of leucovorin.
prodrug with similar activity.

a
Lock
s
Suspend
8
End Bl ock
Item: 3 of 16 ~ 1 • M k -<:J 1>- Jil ~· !:';-~
QIO: 4573 ..L ar Pre v ious Next Lab fli!ltues Notes Calcula t o r

1 thymidylate synthase •

2 - ~ dTMP - ~ Dl A
synthesis.
3
Methotrexate Folic acid analog that Cancer~: leukemias fyelosuppression, "hich is
·4
competitively inhibits (ALL), lymphomas, reversible " it h leuco' orin
•5
dihydrofolate reductase choriocarcinoma, sarcomas. rescue .,

•6 - I dT~IP - I D1 A ~on-neoplastic: ectopic llepatotoxicity.


.7 srnthcsis. pregnane}. medical l\lucositis (cg, mouth ulcers).
·8 abortion (with misoprostol), Pulmonary fibrosis.
.9 rheumatoid arthritis, psoriasis,
IBD, vasculitis.
• 10
a II arc S-phase specific .
• 11

• 12
FA17 p457.2
• 13
Gout drugs
• 14
Chronic gout drugs (preventive)
• 15
Allopurino l Competitive inhibitor of xanthine oxidase. Diet - - Purines Nucleic acids
• 16
~ conversion of hypoxanthine and xanthine to
urate. Also used in lymphoma and leukemia l
Hypoxanthine
to prevent tumor lysis-associated urate
nephropathy. t concentrations of azathioprine
I Xanthine

and 6-.\1P (both normally metabol ized by


t oxidase )
- Allopurino~
xanthine oxidase).
Xanthine
- febuxostat
-. t I •I
·' .
•• . . .
I Xanthine
,•wlA:.c:o

a
Lock
s
Suspend
8
End Bl ock
Item: 3 of 16 ~ 1 • M k -<:J 1>- Jil ~· !:';-~
QIO: 4573 ..L ar Pre v ious Next Lab fli!ltues Notes Calcula t o r

1 •

2 FA17 p 4 57.2

3 Gout drugs

·4 ChroniC gout drugs (preventive)


•5 Allopurinol Competiti,·e inhibitor of xanthine oxidase. Diet - - Purines - - Nucleic acids
•6 I conversion of hypoxanthine and xanthine to
urate. AJso used in lymphoma and leukemia
.7 Hypoxanthine
to pre,·ent tumor lysis-associated urate
·8 Xanthine
nephropathy. t concentrations of at.athioprine
Xanlhi~xidase
.9
• 10
and 6-~ I P (both normally metabolized by
xanthine oxidase).
)
- Allopurinol
febuxostat

• 11 Febuxostat Inhibits xanthine oxidase. ! Xanthine


oxidase

Plasma - - Urate crystals - - Gout


• 12 Peg loticase Recombinant uricase that catalyzes metabolism uric acid deposited
• 13 of uric acid to allantoin (a more water-soluble in JOints
• 14 product).
• 15 Probenecid Inhibits reabsorption of uric acid in proxi mal Tubular

~
conmluted tubule (also inhibits secretion of reabsorption
• 16
penicillin). Can precipitate mic acid ca lc11li. Probenecid and
high-dose salicylates
Acute gout drugs Tubular
-,....---- secretion
:;:
+it----~
NSAIDs Any full-dose 1SAID (eg, naproxen,
indomethacin). Avoid salieylates (may decrease Diuretics and
Urine low-dose saticylates
uric acid excretion, particularly at low doses).
Glucocorticoids OraL intra-articular. or oarenteral.
a
Lock
s
Suspend
8
End Bl ock
urate. Also used in lymphoma and leukemia
2 Hypoxanthine
to pre\·ent tumor lysis- associated urate
3
nephropathy. t concentrations of azathioprine
I Xanthine

·4 and 6-.\lP {both normally metabolized by


t oxidase )
Xanthine - Allopurinol,
•5 xanthine oxidase). - febuxostat
Xanthine
•6 Febuxostat Inhibits xanthine oxidase. OXJdase

.7 Plasma - -+ Urate crystals - -+ Gout


Pegloticase Recombinant uricase that catalyzes metabolism uric acid depoSited
·8 of uric acid to allantoin (a more \\ ater-soluble mJOints
.9 product).
• 10 Probenecid Inhibits reabsorption of uric acid in proxirmll Tubular

~
com·oluted tubule (also inhibits secretion of reabsorptiOn
• 11
penicillin). Can precipitate uric acid calculi. Probenecid and
• 12
high-dose salicylates
Acute gout drugs Tubular
• 13 -,...---- secretion
:;:
~1------~
NSAIDs Any full-dose NSA10 {eg, naproxen,
• 14
indomethacin). Avoid sal icylates (may decrease Diuretics and
• 15 Urine low-dose salicytates
uric acid excretion, particularly at low doses).
• 16
Glucocorticoids Oral, intra-articular, or parenteral.
Colchicine Binds and stabilizes tubulin to inhibit
microtubule polymerization, impairing
neutrophil chemotaxis and degranulation.
Acute and prophylactic value. C l side effects.

a
Lock
s
Suspend
8
End Block
Item: 4 of 16 ~ 1 • M k -<:J 1>- Jil ~· !:';-~
QIO: 3117 ..L ar Pre v ious Next Labfli!llues Not es Calcula t o r

1 •
A 23-year-old athletical ly built woman comes to the physician complaining of multiple red, ring-like lesions
2
on her body, as seen in the image . A carefu l history reveals that she acquired these skin lesions after using
3 poorly cleaned yoga mats at a local gym. The physician prescribes a medicine to clear her rash. After 15 days
.4 of taking the medication, the woman returns to the office. The skin lesions are clearing, but she has noticed that
patches of her skin have become darker than normal.
•5
•6
.7
·8
.9
• 10
• 11

• 12
• 13
• 14
• 15
• 16

Image courtesy of CDC/Or. Lucille K. Georg

Wh ich of the following med ications is this patient most likely taking?
:

a
Lock
s
Suspend
8
End Bl ock
2

3
.4
•5
•6
.7
·8
.9
• 10

• 11
Image courtesy of CDC/Or. Lucille K. Georg
• 12
• 13
Which of the fo llowing medications is this patient most li kely ta king?
• 14
:
• 15
A. Amphote ricin B
• 16
B. Fluconazo le
C. Flucytosine
D. Itraconazole
E. Ketoconazole

a
Lock
s
Suspend
8
End Block
Item: 4 of 16 ~. , . M k <:] t> al ~· ~
QIO: 3117 .l. ar Previous Next Lab 'lifllues Notes Calculator

1
The correct answer is E. 42°/o chose this.
2
This patient has tinea corpo ris (shown in the image in t he stem), a fungal skin infection that is seen in athletes
3
who play d ose-contact sports or use improperly cleaned sports equ ipment. Lesions are pruritic, erythematous,
4 scaly, and demonstrate centra l clearing. Mi ld tinea corporis can be t reated with topica l antifunga ls, but systemic
•5 the rapy, such as ketoconazole, is preferred fo r more extensive skin involvement. Ketoconazole acts by blocki ng
•6 the formation of fungal membrane ste roids. I n addition, the drug inhibits the enzymes desmolase/CYP450scc
and 17-a-hydroxylase. Inhibition of 17-a-hydroxylase contributes to ketoconazole's antiandrogen ic effects,
.7
which incl ude decreased libido, impotence, and gyneco mastia in men . This patient's darken ing skin is a result of
•8 desmolase inhibition. Desmolase is necessary for ad rena l production of testosterone and cortisol. Free cortisol is
•9 responsible for feedback inhibition of the POMC gene, wh ich codes for synthesis of ACTH, lipotropin,
melanocyte-stimu lating hormone, and some endogenous endorphins. With decreased cortiso l levels, this
• 10
feedback inhibition is removed and the POMC gene products are freely transcribed. Excessive melanocyte-
· 11 stimu lating ho rmone can cause increased integumenta ry pigmentation, simi lar to the hype rpigmentation seen in
• 12 Add ison 's disease .
Addison' s disease Gynecomastia Ketoconazole Cortisol Melanocyte-stimulating hormone Testosterone Gene Hyperpigmentation
• 13
Cholesterol side-chain cleavage enzyme Adrenocorticotropic hormone libido Itch Tinea Proopiomelanocortin Hormone Steroid Endogeny (biology) Erythema
• 14
Antifungal Enzyme inhibitor Erectile dysfunction Fungus Infection Anabolic steroid Enzyme Adrenal gland lipotropin
• 15
• 16 A is not correct. 10% chose this .
The patient presents with t inea corporis, a funga l skin infection. Amphotericin B is an antifungal drug used to
treat seve re systemic mycoses. I t disrupts funga l wal l synthesis by binding to ergosterol , a component of the
cell wall. Adverse effects include fever and chi lls, decreased creatinine clearance, hypotension, and anemia.
Amphotericin B Ergosterol Tinea corporis Mycosis Antifungal Cell wall Hypotension Creatinine Fungicide Anemia Renal function Tinea Fungus Skin infection Fever

Infection

ft :_ --&. - - .......... --~ 4ftn' - L - - - &.L:-

6
lock
s
Suspend
0
End Block
Item: 4 of 16 ~. , . M k <:] t> al ~· ~
QIO: 3117 .l. ar Previous Next Lab 'lifllues Notes Calculator

1 Infection

2 B is not correct. 18% chose this .


3 Fluconazole is an antifunga l drug with the same mechanism of action as ketoconazole, but without its adverse
4 endocrine effects . Fl uconazole has good penetration into the cerebrospina l fluid and is used to treat
•5
Cryptococcus neoformans . It is also used to treat Candida infections . Adverse effects include nausea and
vomit ing .
•6 Cryptococcus neoformans Ketoconazole Fluconazole Cerebrospinal fluid Antifungal Fungicide Candida (fungus) Nausea Mechanism of action Vomiting Cryptococcus
.7 Endocrine system
•8
C is not correct . 12% chose this .
•9
Flucytosine is an antifunga l drug used solely in combination with amphotericin B to treat systemic Cryptococcus
• 10 neoformans and system ic Candida infections. Adverse effects include pancytopenia, elevated liver enzyme
· 11 levels, nausea, and vomiting .
Cryptococcus neoformans Amphotericin B Flucytosine Pancytopenia Antifungal Enzyme liver function tests Candida (fungus) Fungicide Nausea liver Vomiting
• 12
• 13 Cryptococcus

• 14 D is not correct. 18% c hose this .


• 15 Itraconazole has the same mechanism of action as ketoconazole, but lacks its endocrine effect s. It is used to
• 16 treat blastomycosis and AIDS- associated histoplasmosis. It has a broader spectrum of act ivity than f luconazo le,
but does not have its CNS penetration . Adverse effects include nausea and vomiting, as well as rash in
immunocomprom ised patients.
Itraconazole Ketoconazole Blastomycosis Histoplasmosis Fluconazole Immunodeficiency Rash Nausea Vomiting Central nervous system Endocrine system

Mechanism of action HIV/AIDS

e _ ...._ _ _ 1 =--·
6
lock
s
Suspend
0
End Block
Item: 4 of 16 ~. , . M k <:] t> al ~· ~
QIO: 3117 .l. ar Previous Next Lab 'lifllues Notes Calculator

2 C is not correct. 12% chose this.


3 Flucytosine is an antifunga l drug used solely in combination with amphotericin B to treat systemic Cryptococcus
neoformans and systemic Candida infections. Adve rse effects include pancytopenia, elevated liver enzyme
4
levels, nausea, and vomiting .
•5 Cryptococcus neoformans Amphotericin B Flucytosine Pancytopenia Antifungal Enzyme liver function tests Candida (fungus) Fungicide Nausea liver Vomiting
•6 Cryptococcus
.7
D is not correct. 18% chose this .
•8
It raconazole has the same mechanism of action as ketoconazole, but lacks its endocrine effects . It is used to
•9 treat blastomycosis and AIDS-associated histoplasmosis. It has a broade r spectrum of activity than f luconazo le,
• 10 but does not have its CNS penetration. Adverse effects include nausea and vomiting, as well as rash in
· 11 immunocompromised patients.
Itraconazole Ketoconazole Blastomycosis Histoplasmosis Fluconazole Immunodeficiency Rash Nausea Vomiting Central nervous system Endocrine system
• 12
Mechanism of action HIV/AIDS
• 13
• 14
• 15 Bottom Line:
• 16 Desmolase inhibition by ketoconazole resu lts in reduced levels of ci rculating ad rena l ste roid hormones. Without
free cortiso l to feedback-inh ibit ACTH, POMC gene products including melanocyte-stimu lating hormone are
freely transcribed; th is leads to the skin darken ing sometimes seen with ketoconazole use.
Ketoconazole Cortisol Melanocyte-stimulating hormone Gene Adrenocorticotropic hormone Cholesterol side-chain cleavage enzyme Steroid hormone Steroid
Proopiomelanocortin Hormone Adrenal gland

6
lock
s
Suspend
0
End Block
Item: 4 of 16 ~ 1 • M k -<:J 1>- Jil ~· !:';-~
QIO: 3117 ..L ar Pre v ious Next Labfli!llues Not es Calcula t o r

1 •

2 FA17 p 148.1
Cutaneous mycoses
3
4
Tinea Tinea is the clinical name given to dermatoph) le (cutaneous fungal) infections. Dermatophytes
(dermatophytes) include Microsporum, Trichophyton, and Epidermophyton. Branching septate hyphae visible on
•5
KOH preparation with blue fungal stain . ssociated with pruritus.
•6
Ttnea capitis Occurs on head, scalp. ssociated '' ith lymphadenopall1}, alopecia, scaling : .
.7
Tinea corporis Occurs on torso. Characterized by eq thematou~ ~eating ring~ ("ringworm..) and central
·8
clearing m. Can be acquired from contact with an infected cat or dog.
.9
Tinea cruris Occurs in inguinal area (!]. Often does not show the central clearing seen in tinea corporis.
• 10
Tinea pedis Three varieties:
• 11
• Interdigital 0 ; most common
• 12 Moccasin distribution 0
• 13 • Vesicular type
• 14 Tinea unguium Onychomycosis; occurs on nails.
• 15 Tinea (pityriasis) Caused by Malassezia spp. (Pityrosporum spp.), a yeast-like fungus (not a dermatop hyte despite
• 16 versicolor being called tinea). Degmdation of lipids produces acids that damage melanocytcs and cause
hypopigmented [!), hypcrpigmcntcd, and/or pink patches. Less pruritic than dermatophytcs.
Can occur any time of yea r, but more common in summer (hot, humid weather). "Spaghetti and
meatballs" appearance on microscopy
Treatment: selenium sulfide, topical and/or oral antifungal medications.
I
-~~- & ...

a
Lock
s
Suspend
8
End Bl ock
Item: 4 of 16 ~ 1 • M k -<:J 1>- Jil ~· !:';-~
QIO: 3117 ..L ar Pre v ious Next Labfli!llues Not es Calcula t o r

1 Can occur any time of yea r, but more common in summer (hot, humid weather). "Spaghetti and •

2 meatballs" appearance on microscopy


Treatment: selenium sulfide, topical and/or oral antifungal medications.
3
4

•5
•6
.7
·8
.9
• 10
• 11

• 12
• 13
• 14
• 15
• 16

FA17 p 195.4
Azoles Clotrimazole, fluconazole, itraconazole, ketoconazole, miconazole, ,·oriconazole, isavuconazole.
MECHANISM Inhibit fungal sterol (ergosterol) svnl hcsis bv inhibiting the cytochrome P-450 enzyme that com·erts

a
Lock Suspend
s 8
End Bl ock
Item: 4 of 16 ~ 1 • M k -<:J 1>- Jil ~· !:';-~
QIO: 3117 ..L ar Pre v ious Next Labfli!llues Not es Calcula t o r

1 •

2 FA17 p 195.4

3
Azoles Clotrimazole, fluconazole, itraconazole, ketoconazole, miconazole, ,·oriconazole, isavuconazole.
4 MECHANISM Inhibit fungal sterol (ergosterol) synthesis by inhibiting the cytochrome P-450 enzyme that com·erts
lanosterol to ergosterol.
•5
•6
CLINICAL USE Local and less serious S}Stemic mrcoses. Fluconazole for chronic suppression of cryptococcal
meningitis in ID patients and candida I infections of all types. ltraconazole for Blastomyces,
.7
Coccidioides, 1/istoplasma. Clotrima.tole and miconawle for topical fungal infections.
·8 Voriconazole for Aspergillus and some Candida. lsavuconazole for serious Aspergillus and
.9 .Yiucorales infections.
• 10 AOVERSEEFFECTS Testosterone synthesis inhibition (gynecomastia, especially with ketoconazole), Ji,·er dysfunction
• 11 (inhibits cytochrome P-450).
• 12
• 13 FA1 7 p 623.2

• 14
Antiandrogens

• 15 Finasteride 5a-reductase inhibitor (l conversion of Testosterone ;a-rcducl•~>c DHT (more potent).


testosterone to DHT). Used for BPH and ma le-
• 16
pattern baldness.
Flutamide onsteroidal competitive inhibitor at androgen
receptors. Used fo r prostate carcinoma.
Ketoconazole Inhibits steroid synthesis (inhibits 17,20
desmolase/17a-hydroxylase). Used in PCOS to reduce androgenic symptoms.
Soironolactone Inhibits steroid bindin!!. 17.20 desmolase/17a- Both can cause gynecomastia and amenorrhea.

a
Lock
s
Suspend
8
End Bl ock
Item: 5 of 16 ~. , . M k <:] t> al ~· ~
QIO: 2473 .l. ar Previous Next lab 'lifllues Notes Calculator

1
A 34-yea r-old man presents to the eme rgency department because of difficu lty walking and extreme pain
2
around his right an kle. Past medica l history is relevant for recent renal transp lantation for kidney failure .
3 Upon examination his mentation is norma l, and he denies any chest pain or shortness of breath . I n addition
4 to ataxia, the patient is found to be unable to plantar flex his right foot. There is also a large, soft, muscular mass
on the proxima l portion of his right calf, and he reports exqu isite tenderness around the right ankle when reflexes
0 5
are tested. Over the course of the examination the man reports that he has been taking a pi ll for "a bug I had."
0 6
7
Wh ich of the fo llowing med ications did this patient most like ly take to treat his infection?
0

0 8
:
0 9
A. Amoxicil lin
0 10
B. Ceftriaxone
· 11
0
12 C. Ciprofloxacin
0
13 D. Erythromycin
0 14
E. Metronidazo le
0 15
0
16
F. Penicil lin G
G. Vancomycin

6
lock
s
Suspend
0
End Block
Item: 5 of 16 ~. , . M k <:] t> al ~· ~
QIO: 2473 .l. ar Previous Next lab 'lifllues Notes Calculator

2 The correct answer is C. 51°/o chose this.


3 Fluoroquinolones have been reported to cause tend init is, particu larly Achilles tend init is. Occasionally, an
affected tendon can rupture, producing a clinica l picture simila r to that described here. People at increased risk
4
for fluoroquinolone-induced tend init is are those >60 yea rs old and those with a history of rena l disease,
5 hemod ialysis, rena l transplant, or long-te rm g lucocorticoid use.
•6 Glucocorticoid Hemodialysis Tendinitis Quinolone Tendon Kidney disease Achilles tendinitis Kidney transplantation

.7 A is not correct. 7°/o chose this.


•8 Adverse effects of amoxicil lin include hype rsensitivity reactions, nausea, vomiting, and dia r rhea .
•9 Amoxicillin Diarrhea Hypersensitivity Nausea Vomiting

• 10 B is not correct. 10% chose this.


· 11 Adverse effects of ceftriaxone include hype rsensitivity reactions, nausea, vomiting, diar rhea, elevated live r
• 12 enzyme leve ls, Clostridium difficile infection, and nephrotoxicty .
Ceftriaxone Enzyme Clostridium difficile colitis Diarrhea liver function tests Clostridium difficile (bacteria) Hypersensitivity Nausea Vomiting liver Clostridium
• 13
Infection
• 14
• 15 Dis not correct. 11% chose this .
• 16 Gastrointestina l irritation is common with eryth romycin via stimu lation of motilin receptors. Hype rsensitivity-
based acute cholestatic hepatit is may be seen with use of eryth romycin estolate. Othe r adverse effects include
skin rashes and eosinoph ilia. Be aware of the risk of deve loping hypertrophic pyloric stenosis if given to infants.
Motilin Erythromycin Pyloric stenosis Cholestasis Stenosis Gastrointestinal tract Human gastrointestinal tract Hepatitis Rash Hypertrophy Pylorus

E is not correct. 9°/o chose this.


Metronidazo le is known fo r its ability to induce a disu lfiram-l ike reaction (flush ing, hypotension, and
tachycardia) with alchol use. Other adverse effects include nausea, vomitina, headache, dizziness, rash,
6
lock
s
Suspend
0
End Block
Item: 5 of 16 ~. , . M k <:] t> al ~· ~
QIO: 2473 .l. ar Previous Next lab 'lifllues Notes Calculator

1
Dis not correct. 11% chose this.
2
Gastrointest ina l irritation is com mon wit h eryth romycin v ia stimu lation of motilin recept ors. Hype rsensit ivity-
3 based acute cholestatic hepatit is may be seen wit h use of erythromycin est olat e. Othe r adverse effect s incl ude
4 skin rashes and eosinoph ilia. Be aware of t he risk of deve loping hypertrophic pyloric stenosis if given to infants.
Motilin Erythromycin Pyloric stenosis Cholestasis Stenosis Gastrointestinal tract Human gastrointestinal tract Hepatitis Rash Hypertrophy Pylorus
5
•6 E is not correct. 9°/o c hose this .
.7 Metronidazo le is known fo r its abilit y to induce a disu lfiram -l ike reaction (flush ing, hypotension, and
•8 tachycardia) with alchol use. Other adverse effects include nausea, vo miting, headache, dizziness, rash,
urticaria, yellow disco loration of t he skin, and optic nerve damage .
•9
Metronidazole Urticaria Optic nerve Hypotension Tachycardia Nausea Dizziness Headache Vomiting Rash Flushing (physiology)
• 10

· 11
F is not correct. 6°/o c hose this.
Not able pen ici llin G side effect s include hypersensit ivit y reactions, hemolyt ic anemia, and diarrhea . Penici llin G
• 12
is given intravenously or intramuscu larly, not orally .
• 13 Penicillin Benzylpenicillin Hemolytic anemia Diarrhea Intravenous therapy Hypersensitivity Intramuscular injection Anemia Hemolysis Adverse drug reaction

• 14 Side effect
• 15
G is not correct. 6°/o chose this .
• 16
A wel l- known but uncommon side effect of rapid infusion of vancomycin is red man synd rome . Vancomycin can
also lead to nephrotoxicity and more common adverse effects such as thrombocyt osis, eosinoph ilia, and
diarrhea .
Vancomycin Eosinophilia Nephrotoxicity Diarrhea Adverse effect Side effect

Bottom Line:
6
lock
s
Suspend
0
End Block
Item: 5 of 16 ~. , . M k <:] t> al ~· ~
QIO: 2473 .l. ar Previous Next lab 'lifllues Notes Calculator
I I .. .. • I •
1
Metronidazo le is known fo r its ability to induce a disu lfiram-l ike reaction (flush ing, hypotension, and
2
tachycardia) with alchol use. Other adverse effects include nausea, vomiting, headache, dizziness, rash,
3 urticaria, yel low disco loration of the skin, and optic nerve damage.
4 Metronidazole Urticaria Optic nerve Hypotension Tachycardia Nausea Dizziness Headache Vomiting Rash Flushing (physiology)

5
F is not correct. 6°/o chose this .
•6 Notable penici llin G side effects include hypersensitivity reactions, hemolytic anemia, and diarrhea . Penici llin G
.7 is given intravenously or intramuscu larly, not oral ly .
Penicillin Benzylpenicillin Hemolytic anemia Diarrhea Intravenous therapy Hypersensitivity Intramuscular injection Anemia Hemolysis Adverse drug reaction
•8
Side effect
•9
• 10 G is not correct. 6°/o chose this .
· 11 A wel l-known but uncommon side effect of rapid infusion of vancomycin is red man synd rome. Vancomycin can
• 12 also lead to nephrotoxicity and more common adverse effects such as thrombocytosis, eosinophilia, and
diarrhea .
• 13
Vancomycin Eosinophilia Nephrotoxicity Diarrhea Adverse effect Side effect
• 14
• 15
• 16 Bottom Line:
On rare occasions fluoroqu inolones cause tendinitis, particu larly Achi lles tendinitis. People at increased risk for
fluoroquino lone-induced tendinitis are those >60 years old and those with a history of renal disease,
hemod ialysis, rena l transplant, or long-te rm g lucocorticoid use.
Glucocorticoid Tendinitis Hemodialysis Quinolone Kidney disease

6
lock
s
Suspend
0
End Block
Item: 5 of 16 ~ 1 • M k -<:J 1>- Jil ~· !:';-~
QIO: 2473 ..L ar Pre v ious Next Lab fli!ltues Not es Calcula t o r

1 •

2 FA17 p 191 .1

3 Fluoroquinolones CiproAoxacin, norAoxacin, )e,oAoxacin, oAoxacin, moxiAoxacin, gemiAoxacin, enoxacin.


4 MECHANISM Inhibit prokaryotic enzymes topoisomerase
5 II (DNA gyrase} and topoisomera~e IV.
•6 Bactericidal. .\lust not be taken" ith antacids.
.7 CLINICAL USE e
Gram rods of urinar~ and G l tracts (including
Pseudomonas}, some gram (f) organisms, otitis
·8
extern a.
.9
ADVERSE EFFECTS Gl upset. superinfections, skin rashes, headache, Fluoroquinolones hurt attachments to your
• 10
dizziness. Less commonly, can cause leg bon e~ .
• 11 cramps and myalgias. Contraindicated in
• 12 pregnant women, nursing mothers, and
• 13 children< 18 years old due to possible damage
• 14 to cartilage. Some may prolong QT interval.
May cause tendonitis or tendon rupture in
• 15
people> 60 years old and in patients taking
• 16
prednisone. CiproAoxacin inhibits cytochrome
P-450.
MECHANISM OF RESISTANCE Chromosome-encoded mutation in Di
gyrase, plasmid-mediated resistance, efAux
pumps.

a
Lock
s
Suspend
8
End Bl ock
Item: 5 of 16 ~ 1 • M k -<:J 1>- Jil ~· !:';-~
QIO: 2473 ..L ar Pre v ious Next Lab fli!ltues Not es Calcula t o r

1 FA17 p 93. 1 •

2 lymph drainage LYMPH NODE CLUSTER AREA OF BODY DRAINED

3 Cervical I lead and neck


4 I Iilar Lungs
5 lediastinal Trachea and esophagus
•6 Axillary Upper limb, breast, skin above umbilicus
.7 Celiac Liver, stomach, spleen, pancreas, upper
·8 duodenum
.9 Superior mesenteric Lo" er duodenum, jejunum, ileum, colon to
• 10 splenic flexure
• 11 Inferior mesenteric Colon from splenic flexure to upper rectum
• 12 Internal iliac Lower rectum to anal canal (above pectinate
• 13 line), bladder, vagina {middle third), cervix,
• 14
prostate

• 15 Para-aortic Testes, ovaries, kidneys, uterus


• 16 Superficial ingui nal Anal canal (below pectinate line), skin below
umbi licus {except popl iteal area), scrotum,
vulva
Popliteal Dorsolateral foot, posterior calf
Right lymphatic duct draim right side of body above diaphragm.
T horacic duct drains everything else into junction of left subcla\·ian and internal jugular veins.

a
Lock
s
Suspend
8
End Bl ock
Item: 6 of 16 ~ 1 • M k -<:J 1>- Jil ~· !:';-~
QIO: 380 6 ..L ar Pre v ious Next Lab fli!ltues Notes Calcula t o r

1 •
A 67-year-old woman with a history of breast cancer comes to her physician's office w ith some shortness of
2
breath and back pain. She takes no current medications. X- ray of the chest shows no evidence of infection,
3 but does revea l a thoracic vertebral fracture. Given the patient's age and gender, the physician obtains a
4 dual energy X-ray absorptiometry (DEXA) scan, wh ich reveals 30% bone loss throughout her spine, humerus, and
femur. The physician prescribes a medication, and a month later the patient is admitted to the emergency
5
department with excruciating stomach pain and episodes of vomiting blood .
•6
.7
Which of the following is most likely the medication her physician prescribed?
·8
:
.9
A. 25-Hydroxyvitamin D
• 10
B. Alendronate
• 11

• 12 C. Denosumab
• 13 D. Estradiol
• 14
E. Tamoxifen
• 15
• 16

a
Lock
s
Suspend
8
End Bl ock
Item: 6 of 16 ~ 1 • M k -<:J 1>- Jil ~· !:';-~
QIO: 380 6 ..L ar Prev ious Next Lab fli!ltues Notes Calculat o r

1 •

2 The correct answer is B. 58°/o chose this.


3 This patient is suffering from osteoporosis, as evidenced by the vertebral
fracture and the bone density loss on DEXA scan. Osteoporosis is
4
characterized by low bone mass and increased bone fragility, leading to an
5 increased risk of fractures, like that shown in the image to the right.
6 Clinically, osteoporosis is defined by a bone mineral density T-score >2.5
0 7 standard deviations below the mean of the ideal population, with values
between 1 and 2.5 standard deviations defined as osteopenia. Risk factors
o8
include being female, being elderly, having had a previous fracture, having
.9 low body weight, smoking, and excess alcohol intake. This patient is
• 10 presenting with evidence of a bleeding ulcer, which can orig inate from either
• 11
the stomach or esophagus. This is most likely an erosive lesion, which is a
possible adverse effect of bisphosphonate therapy (with drugs such as
• 12 alendronate) for osteoporosis . This is why patients are advised to stay upright
• 13 for at least half an hour after taking bisphosphonates to ensure rapid transit
0 14 through the stomach in order to avoid this possib ly serious complication .
Optima l duration of alendronate use has not been determined . For patients at
• 15
low risk for fracture, consider drug discontinuation after 3-5 years of use.
0
16 Other medications that can also cause esophag itis include antibiotics
(clindamycin, tetracycline, and doxycycl ine), potassium chloride, quinid ine,
aspirin and anti-inflammatory agents . Image courtesy of James
Bisphosphonate Bone density Doxycycline Osteoporosis Tetracycline Alendronic acid Osteopenia Esophagus Heilman, MD
Vertebral compression fracture Aspirin Esophagitis Bone fracture Anti-inflammatory Bone Antibiotics Lesion Peptic ulcer Potassium Stomach Quinidine

Adverse effect Tobacco smoking Alcohol Ulcer Dual-energy X-ray absorptiometry

a
Lock
s
Suspend
8
End Bl ock
Item: 6 of 16 ~. , . M k <:] t> al ~· ~
QIO: 3806 .l. ar Previous Next lab 'lifllues Notes Calculator

1
A is not correct. 9°/o chose this.
2 25-Hydroxyvitamin D is often used in conjunction with calcium in the treatment and prevention of osteopo rosis.
3 Vitamin D becomes 1-hydroxylated in the kidney and helps abso rb ca lcium f rom the digestive tract. It does not
4 have the adverse effect of erosive esophagitis.
Osteoporosis Vitamin D Calcium Kidney Esophagitis Gastrointestinal tract Vitamin Adverse effect
5
6 C is not correct. 14% chose this.
.7 Denosumab is a monoclonal antibody that was approved by the US Food and Drug Administration in 2010 for
the treatment of osteoporosis. It targets the recepto r activator of nuclea r factor-K B (RANK) ligand to prevent
•8
the maturation of osteoclasts that reso rb bone. Its adverse effects include increased risk of urinary tract and
•9 respiratory tract infections, cataracts, obstipation, rashes, and joint pain .
• 10 Osteoporosis Food and Drug Administration Monoclonal antibody Antibody ligand Denosumab Osteoclast Constipation Arthralgia Respiratory tract Cataract

· 11 Urinary system Bone Receptor (biochemistry)

• 12 D is not correct. 8°/o chose this.


• 13 Estradiol can be a therapy fo r osteoporosis; however, recent find ings of an increased risk of adve rse
• 14 ca rdiovascu lar events and breast cancer have widely limited its use. Although estrad iol is an effective therapy
• 15
for osteoporosis with many potential adve rse effects, it does not cause erosive esophagitis or gastritis .
Osteoporosis Gastritis Esophagitis Estradiol Breast cancer Circulatory system Cancer
• 16
E is not correct. 11% chose this.
Tamoxifen is selective estrogen recepto r modu lator (SERM) that is used in t he t reatment of women with breast
cancer. It acts by binding estrogen to estrogen receptor-posit ive cells. Another SERM, raloxifene, has been
shown to be just as effective as tamoxifen in breast cancer prophylaxis and has also been approved fo r the
treatment of postmenopausal osteoporosis.
Selective estrogen receptor modulator Index of oncology articles Raloxifene Osteoporosis Tamoxifen Estrogen Estrogen receptor Breast cancer Postmenopausal

6
lock
s
Suspend
0
End Block
Item: 6 of 16 ~. , . M k <:] t> al ~· ~
QIO: 3806 .l. ar Previous Next lab 'lifllues Notes Calculator

1 the maturation of osteoclasts that reso rb bone. Its adverse effects include increased risk of urinary tract and
2
respiratory tract infections, cataracts, obstipation, rashes, and joint pain.
Osteoporosis Food and Drug Administration Monoclonal antibody Antibody ligand Denosumab Osteoclast Constipation Arthralgia Respiratory tract Cataract
3
Urinary system Bone Receptor (biochemistry)
4
5 D is not correct. 8°/o chose this.
6
Estradiol can be a therapy fo r osteoporosis; however, recent find ings of an increased risk of adve rse
ca rdiovascu lar events and breast cancer have widely limited its use. Although estrad iol is an effective therapy
.7
for osteoporosis with many potential adve rse effects, it does not cause erosive esophagitis or gastritis .
•8 Osteoporosis Gastritis Esophagitis Estradiol Breast cancer Circulatory system Cancer

•9
E is not correct. 11% chose this .
• 10
Tamoxifen is selective estrogen recepto r modu lator (SERM) that is used in t he t reatment of women with breast
· 11 cancer. It acts by binding estrogen to estrogen receptor-posit ive cells. Another SERM, raloxifene, has been
• 12 shown to be just as effective as tamoxifen in breast cancer prophylaxis and has also been approved fo r the
treatment of postmenopausal osteoporosis .
• 13
Selective estrogen receptor modulator Index of oncology articles Raloxifene Osteoporosis Tamoxifen Estrogen Estrogen receptor Breast cancer Postmenopausal
• 14
Menopause Receptor (biochemistry) Preventive healthcare Cancer
• 15
• 16
Bottom Line:
Osteoporosis is defined by low bone mass and increased bone frag ility. It is often seen in the elderly,
particularly females, and first-line the rapy usual ly includes bisphosphonates such as alend ronate. One of the
major adve rse effects of t hese drugs is erosive esophag it is.
Osteoporosis Alendronic acid Esophagitis Bisphosphonate

6
lock
s
Suspend
0
End Block
Item: 6 of 16 ~ 1 • M k -<:J 1>- Jil ~· !:';-~
QIO: 380 6 ..L ar Pre v ious Next Lab fli!ltues Notes Calcula t o r

1 •

2 FA17 p 435.2

3 Osteoporosis Trabecular (spongy) and cort ica I bone lose mass Can lead to vertebral compression fractures (fJ,
4 NOITTloll Normal and interconnections despite normal bone sma ll arrows; large arrows show normal-for-age
1ntervenebral lllterverlebral mineralization and lab values (serum Ca 2+ and ,·ertebral body height for comparison)- acute
5 diSC ~ foramen
P0_.3-). back pa in, loss of height, kyphosis. lso can
6
l\ lost commonly due to t bone resorption present with fractures of femoral neck, distal
7
0

related to l estrogen le\·els and old age. radius (Colles fracture).


o8 ~ Can be secondary to drugs (eg, steroids,
.9 alcohol, anticom ulsants, anticoagulants,
• 10 thyroid replacement therapy) or other
Central expai\SIOil Restncted
of inteMrtebral lllterverlebral medical conditions (eg, hyperparathyroidism,
• 11
~ foramen
• 12
d1sc
.. v
. ....,
hyperthyroidism, multiple myeloma,
malabsorption syndromes).
• 13 ~
:=-~oN
-~>....
Diagnosed by a bone mineral density sca n (dual-
0 14 energy x-ray absorptiometry fOF.XA]) with a
• 15 M1ld coml)(ession friKture T-score of s; - 2.5 or by a fragility frc1 cture of
hip or ,-ertebra. Screening recommended in
0
16
women > 65 years old.
Prophylaxis: regular weight-bearing exercise
and adequate Ca2+ and vitamin D intake
throughout adulthood.
Treatment: bisphosphonates, tcriparat ide,
SER.\IIs, rarely calcitonin; denosumab
/ ..,,..."..., ,.'-'.... In n ., ) ., .,. ;h n ...J, . ,..,..,...; .-. r. I) d ' 1L.'I \

a
Lock
s
Suspend
8
End Bl ock
Cenltal txp.1n110n
of inter;ertebra1 •nteNertebral medical conditions (eg, hyperparathyroidism,
d1sc ~foramen
2 h) perthyroidism, multiple myeloma,
3
4
5
_; ._ /3
l>'•ld ~esSIOfl fractuR!
malabsorption syndromes).
Diagnosed by a bone mineral density scan (dual-
energy x-ray absorptiometry [DEXA]) "ith a
T-score of S: -2.5 orb~ a fragili ty fracture of
6
hip or ,·ertebra. creening recommended in
0 7 women > 65 years old.
o8 Prophylaxis: regular weight-bearing exercise
0 9 and adequate Ca1+ and vitamin 0 intake
0 10
throughout adulthood.
Treatment: bisphosphonates, teriparatide,
0
11
SERMs, rarely calcitonin; denosumab
0
12 (monoclonal antibody aga inst RAt Kl .).
0
13
0 14 FA17 p 456.5
• 15 Bisphosphonates Alendronate, ibandronate, risedronate, zolcdronate.
0
16 MECHANISM Pyrophosphate analogs; bi nd hydroxyapatite in bone, inhibiting osteoclast activity.
CLINICALUSE Osteoporosis, hypercalcemia, Paget disease of bone, metastatic bone disease, osteogenesis
imperfecta.
ADVERSE EFFECTS Esophagitis (if ta ken orally, patients are advised to take with water and remain upright for 30
minutes), osteonecrosis of ja", atypical stress fra ctures.

a
Lock
s
Suspend
8
End Block
Item: 7 of 16 ~. , . M k <:] t> al ~· ~
QIO: 4205 .l. ar Previous Next lab 'lifllues Notes Calculator

1
A 27-year-old man is scheduled to undergo hernia repair. On administration of an inha led anesthetic, the
2
patient begins to exper ience severe muscle rigidity, and his temperature rises to 39.4°C ( 102.9°F) . In light
3 of these findings, the anesthesiologist administers another med ication and cools the patient with ice.
4 Subsequently, the patient's temperatu re begins to norma lize and his muscles relax .
5
6 What is the mechan ism of the second drug that was used to treat the patient's adverse reaction from the first
drug ?
.7
•8 :
A. Blocks action potentia l depolar ization from opening voltage-gated ca lcium chan nels
•9
• 10 B. Hy perpolarizes voltage-gated calcium channels on skeletal muscle
· 11 C. Inhibits adenosine t r iphosphate from bi nding to myosin heads
• 12
D. Inhibits the binding of calcium to troponin C
• 13
• 14
E. Inhibits the release of ca lcium from the sarcop lasmic reticu lum
• 15
• 16

6
lock
s
Suspend
0
End Block
Item: 7 of 16 ~. , . M k <:] t> al ~· ~
QIO: 4205 .l. ar Previous Next lab 'lifllues Notes Calculator

1 The correct answer is E. 60°/o chose this.


2 Mal ignant hyperthermia is an autosoma l-dominant genetic disorder affecting ske leta l muscles. It presents as
3 muscu lar rigidity and hyperthermia when an affected patient undergoes sedation via a general anesthetic such
4
as halothane. A ryanod ine receptor-mediated increase in the free ca lcium concentration in this patient's skeleta l
muscle is to blame for this disorder. Dantrolene acts by inhibiting the ryanod ine receptor ca lcium -release
5 channels, thus lowering the intracellular calcium concentration and its ensuing muscle r igidity .
6 Ryanodine receptor Halothane Dantrolene Malignant hyperthermia Hyperthermia General anaesthetic Genetic disorder Anesthesia Sedation Hypertonia

7 Dominance (genetics) Skeletal muscle Intracellular Anesthetic Calcium Cancer Muscle Malignancy

•8
A is not correct. 14% chose this .
•9
Voltage-gated ca lcium channels on the presynaptic membrane open after the action potent ial causes membrane
• 10 depolarization. The influx of ca lcium induces neurotransmitter release. Dantrolene plays no direct role in this
· 11 process.
Action potential Dantrolene Neurotransmitter Depolarization Voltage-dependent calcium channel Calcium Calcium channel Chemical synapse Synapse
• 12
Voltage-gated ion channel Cell membrane Active zone Synaptic vesicle
• 13
• 14 B is not correct. 12% chose this .
• 15 Dantrolene does not aff ect the voltage-gated channels of ske letal muscles. It acts after this event and prevents
• 16
the release of calcium from the sarcoplasmic reticulum .
Dantrolene Sarcoplasmic reticulum Skeletal muscle Calcium

C is not correct. 6°/o chose this.


Adenosine t r iphosphate binds to the myosin head in skeletal muscle and releases it from the actin filament.
Dantrolene has no direct ro le in inhibiting this process.
Adenosine triphosphate Dantrolene Myosin Actin Skeletal muscle Adenosine Microfilament Muscle

6
lock
s
Suspend
0
End Block
Item: 7 of 16 ~. , . M k <:] t> al ~· ~
QIO: 4205 .l. ar Previous Next lab 'lifllues Notes Calculator
• •
1
Action potential Dantrolene Neurotransmitter Depolarization Voltage-dependent calcium channel Calcium Calcium channel Chemical synapse Synapse
2
Voltage-gated ion channel Cell membrane Active zone Synaptic vesicle
3
4
B is not correct. 12% chose this.
Dantrolene does not affect the voltage-gated channels of ske letal muscles. It acts after this event and prevents
5
the release of calcium from the sarcoplasmic reticulum.
6 Dantrolene Sarcoplasmic reticulum Skeletal muscle Calcium

7
C is not correct. 6°/o chose this .
•8
Adenosine t r iphosphate binds to the myosin head in skeletal muscle and releases it from the actin filament .
•9 Dantrolene has no direct ro le in inhibiting this process.
• 10 Adenosine triphosphate Dantrolene Myosin Actin Skeletal muscle Adenosine Microfilament Muscle

· 11 D is not correct. 8°/o chose this .


• 12 The bind ing of calcium to t roponin C causes a confor mationa l change in skeletal muscle. This causes
• 13 tropomyosin to move out of the way, al lowing actin/ myosin cycl ing. Dantro lene may ind irectly inhibit the
• 14 binding of calcium via tropon in C by reducing the overal l level of calcium released by the sarcoplasmic
reticu lum. However, it does not directly inhibit the bind ing of calcium to t roponin C.
• 15
Sarcoplasmic reticulum Tropomyosin Troponin Skeletal muscle Calcium TPMl Troponin c type 1 Troponin c Muscle
• 16

Bottom Line:
Dantrolene exerts its effect by preventing the release of calcium from the sarcoplasmic reticulum of skeletal
muscle, thereby preventi ng muscle contraction.
Dantrolene Sarcoplasmic reticulum Skeletal muscle Muscle contraction Calcium Muscle

6
lock
s
Suspend
0
End Block
Item: 7 of 16 ~ 1 • M k -<:J 1>- Jil ~· !:';-~
QIO: 420 5 ..L ar Pre v ious Next Lab fli!ltues Not es Calcula t o r

1 •
FA17 p 520.2
2 Oantrolene
3 MECHANISM Pre,·ents release of Ca 2+ from the sarcoplasmic reticulum of skeletal muscle by binding to the
4 ryanodine receptor.
5 CliNICAL USE lalignant hyperthermia and neuroleptic malignant S} ndrome (a toxicity of antipsychotic drugs).
6
7
FA17 p433.1
·8
Muscle conduction to T-tubules are extensions of plasma membrane juxtaposed" ith terminal cisternae of the
.9
contraction sarcoplasmic reticulum, allowing for coordinated contraction of muscles.
• 10
Dihydropyridine receptor In skeletal muscle, I T-tubulc + 2 terminal cisternae= triad.
• 11 I·,IUIIlllle membrane In cardiac muscle, 1 T-tubule + I terminal cisterna = drad .
• 12 I. Action potential depolarization opens presynaptic \Oitage-gated Ca2+ channels, inducing
• 13 neurotransmitter release.
2. Postsynaptic ligand binding leads to muscle cell depolarization in the motor end plate .
• 14
Ryanodine Sarcoplasmic 3. Depolarization travels along muscle cell and down the T-tubule.
• 15 receptor reticulum ~
4. Depolarization of the voltage-sensit ive dihydropyridine receptor, mechanically coupled to the
• 16 ryanodine receptor on the sarcoplasm ic reticulum, induces a conformational change in both
receptors, causing Ca 2+ release from sa rcoplasmic reticulum.
5. Released Ca 2+ binds to troponin C, causing a conformational change that moves tropomyosin
out of the myosin-binding grom e on act in filaments.
6. Myosin releases bound ADP and P, ..... displacement of mrosin on the actin filament (power
stroke). Contraction results in shortening of II and I bands and between Z lines (I liZ
shrinkage), but the r\ band remains the same length (.\ band is _\!ways the same length) r.J_

a
Lock
s
Suspend
8
End Bl ock
Item: 7 of 16 ~ 1 • M k -<:J 1>- Jil ~· !:';-~
QIO: 420 5 ..L ar Pre v ious Next Lab fli!ltues Not es Calcula t o r

1 receptor •
reticulum eJ 4. Depolarization of the vohage-sensil j,·e dihydropyridine receptor, mechanically coupled to I he
2 ryanodine receptor on the sa rcoplasm ic reticulum, induces a confo rmational change in both
3 receptors, causing Ca 2+ release from sarcoplasmic reticulum.
4 5. Released Ca 2+ binds to lroponin C, causing a conformational change that moves tropomyosin
out of the myosin-binding groO\e on actin fi laments.
5
6. \ lyosin releases bound ADP and P, - displacement of myosin on the actin fil ament {power
6
stroke). Cont raction rcsuhs in shortening of II and I bands and bel\\een Z lines (I liZ
7 shrinkage), but the A band remains I he same length (A band is .\]ways tl1e same length)
·8 7. Binding of a new ATP molecule causes detachment of myosin head from actin filament.
.9 Hydrolysis of bound ATP - ADP, myosin head adopts high-energy position ("cocked") for the
• 10
next contraction cycle.

• 11

• 12
• 13
• 14
• 15
• 16

Myofibrtl

.;.;;ba
.__.;.;;A d ll I band
;;;;n;.;___, I
u

a
Lock
s
Suspend
8
End Bl ock
2

3
4
5
Sarcomere
6
A band
7 u
Hband
·8
.9
• 10 FA17 p 519.1
• 11 Inhaled anesthetics DesAurane, halothane, cn Aumnc, isoAumnc, sevoAuranc, methoxyflurane, l 20 .
• 12 MECHANISM Mechanism unknO\\ n.
• 13 EFFECTS Myocardial depression, respiratory depression, nausea/emesis, t cerebral blood Aow U cerebral
• 14 metabolic demand).
• 15 ADVERSEEFFECTS Hepatotoxicity (halothane), nephrotoxicity (mcthoxyAnranc), proconvulsant (cnAurane,
• 16 epileptogenic), expansion of trapped gas in a body cavity (N 20 ).
Malignant hyperthermia- rare, life.threatening condition in which inhaled anesthetics or
succinylcholine induce fever and ~cvc rc muscle contractions. Susceptibility is often inherited as
autosomal dominant with nuiable penelrance. Mutations in voltage-sensitive ryanodine receptor
cause t Ca 1+ release from sarcoplasm ic reticulum. Treatment: dantrolene (a ryanodine recept·or
antagonist).

a
Lock
s
Suspend
8
End Block
Item: 9 of 16 ~ 1 • M k -<:J 1>- Jil ~· !:';-~
QIO: 1449 ..L ar Pre v ious Next Lab fli!ltues Not es Calcula t o r

1 •
A 60-year-old otherwise healthy man has a remote history of gastritis associat ed w ith a med ication
2
prescribed for his osteoarthrit is. Recently, he started experiencing more joint pains whi le perfor ming da ily
3 activities. He asks his family physician for a pain medication to manage his osteoarthritis.
4
5 Which of the following analgesics should be prescribed?
6 :
7 A. Aspir in
·8 B. Celecoxib
.9
C. Diclofenac
• 10
• 11 D. Ibuprofen
• 12 E. Indomethacin
• 13
• 14
• 15
• 16

a
Lock
s
Suspend
8
End Bl ock
Item: 9 of 16 ~. , . M k <:] t> al ~· ~
QIO: 1449 .l. ar Previous Next lab 'lifllues Notes Calculator

2 The correct answer is B. 66°/o chose this.


3
Celecoxib acts by selective ly inh ibiting cyclooxygenase-2 (COX-2), an enzyme responsible for the conversion of
arach idonic acid to prostag land in G2 • This is the first step in the synthesis of the prostagland ins and
4
thromboxanes invo lved in the inflammatory response. Due to its selectivity for COX-2, celecoxib has a
5 decreased r isk of causing adverse gastrointestinal (GI) events such as bleeding and ulcer compared to standard
6 nonse lective nonsteroidal anti-inflammatory drugs (NSAIDs) . Due to recent controversy regarding an increased
r isk of cardiovascu lar events with COX-2 inhibitors, they are prescribed with caution .
7
Prostaglandin Celecoxib Arachidonic acid Prostaglandin-endoperoxide synthase 2 Enzyme Anti-inflammatory Thromboxane Inflammation
•8
Nonsteroidal anti-inflammatory drug COX-2 inhibitor Peptic ulcer Circulatory system Human gastrointestinal tract Gastrointestinal tract
9
• 10 A is not correct. 7°/o chose this .
Aspirin acts by irreversibly inh ibiting both COX-1 and COX-2, which prevents the production of prostaglandins
· 11
that cause inflammation and pain. Clinica lly, it is used as an analgesic, an antipyretic, and an antiplatelet drug .
• 12 However, because aspir in inhibits COX-1, which produces prostag landins that are protective for the gastric
• 13 mucosa, it has the potential to cause adverse GI events . Aspirin would be a poor choice in th is patient,
• 14 particular ly due to his past history of NSAID-induced gastritis .
Antiplatelet drug Antipyretic Analgesic Aspirin Gastritis PTGSl Gastric mucosa Prostaglandin Inflammation Prostaglandin-endoperoxide synthase 2
• 15
Mucous membrane
• 16
C is not correct. 8°/o chose this.
Diclofenac, a nonse lective COX inhibitor with a mode of action similar to that of aspirin, would be
contra indicated in th is patient because of its lack of selectivity in inh ibiting the COX enzyme. Topical diclofenac
may intu it ive ly seem like an appropriate choice; however, it carries the same gastrointestinal side-effect
warn ings as ora l diclofenac. Between 6% and 20% of the topical dose reaches systemic circu lation . Topica l
diclofenac has fewer GI side effects than oral diclofenac; however, celecoxib is the best answer choice .

6
lock
s
Suspend
0
End Block
Item: 9 of 16 ~. , . M k <:] t> al ~· ~
QIO: 1449 .l. ar Previous Next lab 'lifllues Notes Calculator

1
Diclofenac, a nonse lective COX inhibitor with a mode of action similar to that of aspirin, would be
2 contra indicated in th is patient because of its lack of selectivity in inh ibiting the COX enzyme. Topical diclofenac
3 may intu it ive ly seem like an appropriate choice; however, it carries the same gastrointestinal side-effect
warn ings as ora l diclofenac. Between 6% and 20% of the topical dose reaches systemic circu lation. Topica l
4
diclofenac has fewer GI side effects than oral diclofenac; however, celecoxib is the best answer choice.
5 Celecoxib Oiclofenac Aspirin Enzyme Circulatory system COX-2 inhibitor Contraindication Enzyme inhibitor Cyclooxygenase Adverse drug reaction

6 Human gastrointestinal tract Gastrointestinal tract Side effect


7
D is not correct. 10% chose this .
•8
I buprofen is a reversible and nonse lective inh ibitor of COX. I t wou ld increase this patient's risk of adverse GI
9 events such as gastrointestinal bleed ing.
• 10 Ibuprofen Gastrointestinal bleeding Functional selectivity Human gastrointestinal tract Gastrointestinal tract Enzyme inhibitor Cyclooxygenase

· 11 E is not correct. 9°/o chose this .


• 12 I ndomethacin, a competitive antagonist of both COX- 1 and COX- 2, is a NSAID that is considered to be more
• 13 potent than aspir in. I t wou ld be inappropriate due to its inhibit ion of COX-1 and thus decreased production of
• 14 prostag landins that are protective for the gastric mucosa .
Nonsteroidal anti-inflammatory drug Indometacin Aspirin Prostaglandin PTGSl Gastric mucosa Receptor antagonist Prostaglandin-endoperoxide synthase 2
• 15
Mucous membrane
• 16

Bottom Line:
Celecoxib is the on ly remaining COX- 2 inhibitor on the ma r ket, and it is more appropriate for treating arthritis
in people who are sensitive to the adverse GI eff ects of NSAIDs. Extra care must be taken in a patient with a
previous history of GI bleed ing.
Celecoxib Gastrointestinal bleedina Nonsteroidal anti-inflammatorv drua COX-2 inhibitor Prostaalandin-endooeroxide svnthase 2 Arthritis Enzvme inhibitor

6
lock
s
Suspend
0
End Block
Item: 9 of 16 ~ 1 • M k -<:J 1>- Jil ~· !:';-~
QIO: 1449 ..L ar Pre v ious Next Lab fli!ltues Not es Calcula t o r

1 •
FA17 p456.2
2 Celecoxib
3 MECHANISM Reversibly and ~elect ively inhibits the C)clooxygcnasc (COX) isoform 2 ("Sclccoxib"), which is
4 found in inflammatory cells and vascular endothelium and mediates inflammation and pain;
5 spares COX-I, which helps maintain gastric mucosa. Thus, does not hm·e the corrosi,·c effects
6 of other 'SAIDs on the Gllining. Spares platelet fu nction as TXA2 production is dependent on
C OX-1.
7
CLINICAL USE Rheumatoid arthritis, osteoarthritis.
·8
9
ADVERSE EFFECTS t risk of thrombosis. Sulfa allergy.
• 10
FA17 p 456.1
• 11
Aspirin
• 12
MECHANISM !':SAID that irreversibly inhibits cyclooxygcuasc (both COX-I and COX-2) by covalent acctylatiou
• 13
-+ ' synthesis ofTXA and prostaglandins. t bleeding time.
2 o effect on PT, PTT. Effect lasts
• 14
until new platelets are produced.
• 15
CLINICAL USE Low dose(< 300 mg/day): ' platelet <~ ggrega l io n . lutcrmcdialc dose (300-2400 mg/day): antipyretic
• 16 and analgesic. High dose (2400-4000 mg/day): anti-inflammatory.
ADVERSE EFFECTS Gastric ulceration, tinn itus (CI VIII). Chronic 11se can lead to acute renal failure, interstitial
nephritis, Gl bleeding. Risk of RC)C syndrome in children treated with aspirin for viral infection.
Toxic doses cause respiratory alkalosis early, but transitions to mixed metabolic acidosis-respiratory
alkalosis.

a
Lock
s
Suspend
8
End Bl ock
Item: 9 of 16 ~ 1 • M k -<:J 1>- Jil ~· !:';-~
QIO: 1449 ..L ar Pre v ious Next Lab fli!ltues Not es Calcula t o r

1 •

2 FA17 p 456.1
Aspirin
3
4
MECHANISM 1'\SAID that irreversibly inhibits cycloox)genase (both COX-I and COX-2) by co,·alenl acetylation
..... l synthesis ofTXA 2 and prostaglandins. t bleeding time. J o effect on PT, PTT. Effect lasts
5
until new platelets are produced.
6
CliNICAL USE Lo" dose (< 300 mg/day): l platelet aggregation. Intermediate dose (300-2400 mg/da)): antip)retic
7
and analgesic. High dose (2400-4000 mg/day): anli-inAammatory.
·8
ADVERSE EFF£CTS Gastric ulceration, tinnitus (CN VII I). Chronic use can lead to acute renal failure, interstitial
9 nephritis, CI bleeding. Risk of Re)C S) ndrome in children treated with aspirin for' ira I infection.
• 10 Toxic doses cause respiratory alkalosis earl), but transitions to mixed metabolic acidosis-respiratory
• 11 alka los is.
• 12
• 13 FA17 p456.3
• 14 Nonsteroidal Ibuprofen, naproxcn, indomethacin, ketorolac, diclofcnac, meloxicam, piroxicam.
• 15 anti-inflammatory
• 16
drugs
MECHANISM Reversibly inh ibit cyclooxygcnase (both COX-I and COX-2). Block prostaglandin synthesis.
CLINICAL USE Antipyretic, analgesic, anti-in Aammatory. Indomethacin is used to close a PDA.
ADVERSE EFFECTS Interstitial nephritis, gastric ulcer (prostaglandins protect gastric mucosa), renal ischemia
(prostaglandins ,·asodilate afferent arteriole), aplastic anemia.

a
Lock
s
Suspend
8
End Bl ock
Item: 10 of 16 ~ 1 • M k -<:J 1>- Jil ~· !:';-~
QIO: 3887 ..L ar Pre v ious Next Lab fli!ltues Not es Calcula t o r

2
A 22-year-old med ica l student has a positive purified protein derivative test. Results of her chest x-ray show IAA] •
no signs of active disease. For treatment of latent tuberculosis, she begins a 9-month-long medication
3 reg imen. Two months into treatment, she complains of the sudden onset of fever, rash, and swelling of her
4 joints.
5
6 What is the intended mechanism of the medication that caused these adverse effects?
7 :

·8 A. Decreases synthesis of folate


9 B. Decreases synthesis of messenger RNA
• 10
C. Decreases synthesis of mycolic acids
• 11
D. Decreases synthesis of peptidoglycan
• 12
• 13 E. Increases synthesis of cGMP
• 14
• 15
• 16

a
Lock
s
Suspend
8
End Bl ock
Item: 10 of 16 ~ 1 • M k -<:J 1>- Jil ~· !:';-~
QIO: 3887 ..L ar Pre v ious Next Lab fli!ltues Not es Calcula t o r

2
A 22-year-old med ica l student has a positive purified protein derivative test. Results of her chest x-ray show IAA] •
no signs of active disease. For treatment of latent tuberculosis, she begins a 9-month-long medication
3 reg imen. Two months into treatment, she complains of the sudden onset of fever, rash, and swelling of her
4 joints.
5
6 What is the intended mechanism of the medication that caused these adverse effects?
7 :

·8 A. Decreases synthesis of folate


9 B. Decreases synthesis of messenger RNA
• 10
C. Decreases synthesis of mycolic acids
• 11
D. Decreases synthesis of peptidoglycan
• 12
• 13 E. Increases synthesis of cGMP
• 14
• 15
• 16

a
Lock
s
Suspend
8
End Bl ock
Item: 10 of 16 ~. I • M k <:] t> al ~· ~
QIO: 3887 .l. ar Previous Next lab 'lifllues Notes Calculator

2
The correct answer is C. 64°/o chose this.
This student has a positive purified protein derivative (PPD) test, signifying possible exposu re to tubercu losis
3
(TB). To decrease her risk of developing an active TB infection, a prolonged regimen of isoniazid is ind icated.
4 I soniazid causes a decrease in the synthesis of myco lic acids that make up the unique cell envelope of
5 Mycobacterium tuberculosis. When a 9-month regimen for TB is chosen, it typical ly consists of isoniazid alone.
6 Certain combined drug regimens allow for shorter treatment times.
7 One possible adverse effect of taking isoniazid is a lupus-like syndrome. Drug-induced lupus is characterized
•8
by an abrupt onset of symptoms, wh ich may incl ude fever, arthritis, pleura l pericarditis, and rash . Lab studies
may reveal development of anti- histone antibod ies. Other medications that can cause drug-induced lupus
9 syndrome include sulfa d rugs, phenytoin, procainamide, hyd ralazine, and penicil lamine.
10
Procainamide Isoniazid Hydralazine Phenytoin Mycobacterium tuberculosis Penicillamine Tuberculosis Pericarditis Sulfonamide (medicine) Anti-histone antibodies
· 11
Protein lupus erythematosus Antibody Pleural cavity Mycolic acid Mantoux test Mycobacterium Systemic lupus erythematosus Arthritis Fever Infection Rash
• 12
Adverse effect
• 13
• 14
A is not correct. 9°/o chose this .
• 15
Dapsone is used in the treatment of Hansen disease caused by Mycobacterium leprae . It disrupts the synthesis
of folate in a method similar to that of su lfonamides. Other inhibitors of folate metabolism such as
• 16 trimethoprim-sulfamethoxazole are used as prophylactic treatment against pneumocystis pneumonia in
immunocompromised patients. Neither of these medications causes a lupus-l ike syndro me.
Mycobacterium leprae Pneumocystis pneumonia Dapsone Trimethoprim/sulfamethoxazole Immunodeficiency Folic acid Pneumonia Sulfonamide (medicine) leprosy

Mycobacterium Sulfonamide Metabolism Preventive healthcare

B is not correct. 16% chose this.


Rifampin is used to treat tube rculosis th rough suppression of RNA synthesis by inhibiting bacterial DNA-

6
lock
s
Suspend
0
End Block
Item: 10 of 16 ~. I • M k <:] t> al ~· ~
QIO: 3887 .l. ar Previous Next lab 'lifllues Notes Calculator

1
B is not correct. 16% chose this.
2
Rifampin is used to treat tubercu losis through suppression of RNA synthesis by inhibiting bacter ial DNA-
3 dependent RNA polymerase. I t induces the cytochrome P-450 enzyme system, and may cause tears and urine
4 to turn orange in co lor, but it is not known to cause a lupus-like synd rome. Furthermore, the question states
5
that the patient was taking a 9-month cou rse of prophylactic therapy; there are different options but the 9
month du ration of treatment is typica lly for isoniazid alone.
6 Isoniazid Rifampicin Tuberculosis Enzyme Cytochrome P450 Transcription (genetics) Urine Preventive healthcare RNA polymerase RNA
7
D is not correct. 7°/o chose this .
•8
I nhibitors of peptidog lycan synthesis include pen ici llins, cephalosporins, monobactams, carbapenems, and
9 vancomycin. These antibiotics are not used as prophylaxis against TB, and are not associated with a lupus-like
10 synd rome .
Peptidoglycan Vancomycin Monobactam Cephalosporin Carbapenem Penicillin Antibiotics Preventive healthcare
· 11
• 12 E is not correct. 4°/o chose this .
• 13 Hyd ralazine causes vasod ilation th rough increased production of cGMP and may cause a lupus-l ike synd rome .
• 14 However, it wou ld not be used to treat TB. Other drugs that also m ay cause a lupus-like synd rome are
proca inamide and phenytoin .
• 15
Procainamide Hydralazine Phenytoin Vasodilation
• 16

Bottom Line:
I soniazid is an antimycobacterial medication that acts by inhibiting myco lic acid synthesis. A lupus-like
synd rome is an uncommon but possible side effect.
Isoniazid Mycolic acid

6
lock
s
Suspend
0
End Block
Item: 10 of 16 ~ 1 • M k -<:J 1>- Jil ~· !:';-~
QIO: 3887 ..L ar Pre v ious Next Lab fli!ltues Not es Calcula t o r

1 •
FA17 p 193.1
2
Isoniazid
3
MECHANISM ~ synthesis of mycolic acids. Bacterial catalase-
4
peroxidase (encoded by KatG) needed to
5 com·ert IN I-I to acti,·e metabolite.
6 CliNICAL USE lycobacterium tuberculosis. The only agent Different INH half-liYes in fast ,.s slow
7 used as solo prophylaxis against TB. Also used acetdators.
·8 as monotherapy for latent TB.
9 ADVERSE EFFECTS Hepatotoxicity, P-450 inhibition, drug-induced '""'' ' Injures ~eurons and l lepatocytes.
10 SLE, anion gap metabolic acidosis,' itamin
B6 deficiency (peripheral neuropathy,
• 11
side roblastic anemia). dminister with
• 12
pyridoxine (8 6) .
• 13
MECHANISM OF RESISTANCE Mutations leading to underexpression of KatC.
• 14
• 15
FA17 p 241 .2
• 16 Drug reactions- musculoskeletal/skin/connective tissue
DRUG REACTION CAUSAL AGENTS NOTES
Fat redistribution P rotease inhibitors, G lucocorticoids Fat PiG
Gingival hyperplasia Phenytoin, Ca 2+ channel blockers, cyclosporine
Hyperuricemia (gout) Pyrazinamide, T hiazides, Furosemide, '\ iacin, Pain ful Tophi and Feet l\eed C are
C yclosporine

a
Lock
s
Suspend
8
End Bl ock
Item: 10 of 16 ~. I • M k <:] t> al ~· ~
QIO: 3887 .l. ar Previous Next lab 'lifllues Notes Calculator

2 FA17 p 24 1.2
3 Drug reactions-musculoskeletal/skin/connective tissue
4 DRUGREACTION CAUSAL AGENTS NOTES

5 Fat redistribution P rotease inhibitors, G lucocorticoids Fat PiG


6 Gingival hype rplasia Phenytoin, Ca 2+ channel blockers, cyclosporine
7 Hyperuricemia (gout) Pyrazi namide, T hiazides, Furosemide, Niacin, Painful 'lophi and Feet Need C are
•8 C yclosporine
9 Myopathy Statins, fibratcs, n iacin, colchicine, daptomycin,
10 hydroxychloroquinc, interferon-a,
penicillamine, glucocorticoids
· 11
Osteopo rosis Corticosteroids, depot meclroxyprogesterone
• 12
acetate, GnRH agonists, aromatase inhibitors,
• 13
anticonvulsants, heparin
• 14
Photosensitivity Sulfonamides, Amiodarone, Tetracyclines, 5-F U SAT For Photo
• 15
Rash (Stevens- Anti-epileptic drugs {especially lamotrigine), Steven Johnson has ep ileptic allergy to sulfa
• 16
Johnson syndrome) allopurinol, sulfa d rugs, penicillin drugs and penicillin
SLE-Iike syndrome Sulfa drugs, Ilydralazine, Isoniazid, I laving lupus is "SIIIPP-E"
P rocainamicle, Phenytoin, E tanercepl
Teeth discoloration Tetracyclines Tccth racycl ines
Tendonitis, tendon Fluoroqu inolones
rupture, and

6
lock
s
Suspend
0
End Block
Item: 10 of 16 ~ 1 • M k -<:J 1>- Jil ~· !:';-~
QIO: 3887 ..L ar Pre v ious Next Lab fli!ltues Not es Calcula t o r

1 Rash (Stevens- Anti-epileptic dmgs (especially lamol rigi ne), Ste\Cn Johnson has epileptic allergy to mlfa •

2 Johnson syndrome) allopurinol, sulfa drugs, penicillin drugs and penicillin


3 SLE-Iike syndrome Sulfa drugs, Hydralazine, Isoniazid, llm·ing lupus is "SIIIPP-E"
4 Procainamide, Phenytoin, Etancrcept
5 Teeth discoloration Tctracycl ines Tccthracyclincs
6 Tendonitis, tendon F'luoroq u inolones
rupture, and
7
cartilage damage
·8
9
10 FA17 p443.1

• 11
Systemic lupus erythematosus
• 12 SYMPTOMS Classic presentation: rash, joint pain, and fc\·er, RASII OR PAIN:
most commonly in a female of reproductive Rash (malar rJ or discoid : )
• 13
age and African- merican descent. Arthritis (nonerosi\·e)
• 14
Libman-Sacks Endocarditis- non bacteria I, Serositis
• 15 verrucous thrombi usually on mitral or aortic l lcmalologic disorders (eg, cytopcnias)
• 16 valve and can be present on either surface of O ral/nasopharyngeal ulcers
the \·alve {but usually on undersurface) (LSE Renal disease
in SLE). Photosensitivity
Lupus nephritis (glomerular deposition of anti- Antinuclear antibodies
Dt A immune complexes) can be nephritic Immunologic disorder (anti-dsDt A, anti-Sm.
~
'
Bj . or nephrotic (hematuria or proteinuria). ani iphospholipid)
l~
0 I

I Most common and severe type is diffuse l\eurologic disorders (eg, seizures, pS)Chosis)

a
Lock
s
Suspend
8
End Bl ock
va can present on
2
the ,·ah-e (but usually on undersurface) (LSE
in SLE). Photosensitivity
3
Lupus nephritis (glomerular deposition of anti- Antinuclear antibodies
4 DNA immune complexes) can be nephritic Immunologic disorder (anti-dsD1 A, anti- m,
5 or nephrotic (hematuria or proteinuria). antiphospholipid)
6 Most common and severe type is diffuse :\eurologic disorders (eg, seizures, pS) chosis)
7
proliferative.
Common causes of death in SI.E:
·8
Cardio,·ascular disease
9 • Infections
10 • Renal disease
• 11 FINDINGS Antinuclear antibodies (ANA) Sensitive, not specific
• 12 Anti-dsD antibodies Specific, poor prognosis (renal disease)
• 13
Anti-Smith antibodies Specific, not prognostic (directed against
• 14 snR IPs)
• 15 Anti histone antibodies Sensitive for drug-induced lupus (eg,
• 16 hydra lazinc, proca inamidc)
' C3, C4, and CH;o due to immune complex
formation.
TREATMENT !SAl Ds, steroids, immunosuppressants,
hydroxych loroquine.

a
Lock
s
Suspend
8
End Block
Item: 11 of 16 ~. , . M k <:] t> al ~· ~
QIO: 2008 .l. ar Previous Next lab 'lifllues Notes Calculator

1
A 60-year-old man presents with severe pain in his mouth for the past several days. Local ized swe lling, ~~AI
2
exposed bone, erythema, and a puru lent discharge are noted on intraora l exam ination, and he is diagnosed
3 with osteonecrosis of the maxilla. The patient states that he had a mola r tooth extracted four years ago, and
4 that the site never hea led properly. I n add it ion, the patient is being treated for mu lt iple myeloma, which was
diagnosed 11 months ago .
5
6
Wh ich of the fo llowing dr ug classes may have contr ibuted to the deve lopment of the patient 's osteonecrosis?
7
•8 :
A. Angiotensin converting enzyme inh ibitors
9
10 B. Bisphosphonates
• 11 C. Cephalosporins
• 12
D . Osmotic diuretics
• 13
• 14
E. Vinca al kaloids
• 15
• 16

6
lock
s
Suspend
0
End Block
Item: 11 of 16 ~. , . M k <:] t> al ~· ~
QIO: 2008 .l. ar Previous Next lab 'lifllues Notes Calculator

1
The correct answer is B. 63°/o chose this.
2
Osteonecrosis of the jaw (avascu lar necrosis of the jaw) has been associated with dental extraction, loca l
3
infection, pathologic fractu re of the jaw, and ch ronic bisphosphonate therapy. This patient likely developed
4 osteonecrosis due to the combination of both his tooth extraction and bisphosphonate use. Bisphosphonates,
5 such as alendronate and r ised ronate, are used to treat metastatic bone diseases and osteoporosis. Multiple
myeloma causes bone destruction due to increased osteoclast activity. Bisphosphonates decrease pain and
6
fractu res by reducing the number and activity of osteoclasts and inh ibiting bone resorption; they are indicated
7 for the t reatment of any multiple mye loma patient with ske leta l lesions. Although it is unable to cu re existing
•8 lesions, bisphosphonate the rapy wi ll help prevent deve lopment of new lesions. Gastrointestinal toxicity and
9 hypocalcemia have also been reported in patients ta king bisphosphonates.
Osteoclast Bisphosphonate Dental extraction Risedronic acid Pathologic fracture Osteoporosis Hypocalcaemia Multiple myeloma Alendronic acid Avascular necrosis
10
Osteonecrosis of the jaw Bone resorption Metastasis Necrosis Bone disease Pathology Gastrointestinal tract Fracture Infection Bone fracture Jaw Bone
11

• 12 A is not correct. 4°/o chose this .


• 13 Angiotensin converting enzyme (ACE) inh ibitors, such as enalapril, inhibit angiotensin-converting enzyme and
are util ized in the t reatment of hypertension. ACE inh ibitor toxicity can include cough, angioedema, proteinu ria,
• 14
taste changes, rash, and hyper kalemia. Osteonecrosis is not associated with use of ACE inhibitors .
• 15 ACE inhibitor Enalapril Hyperkalemia Angiotensin-converting enzyme Proteinuria Angioedema Enzyme Angiotensin Hypertension Cough Rash Enzyme inhibitor

• 16
C is not correct. 9°/o chose this.
Cephalosporins are ~-lactam antibiotics used to t reat infections caused by gram-positive and gram-negative
bacte ria . The side effects associated with cepha lospo rins are relatively non-specific and may incl ude nausea,
vomiting, diarrhea and a hypersensitivity rash. Use of cephalospor ins does not cause osteonecrosis.
Gram-negative bacteria Gram-positive bacteria Diarrhea Hypersensitivity Nausea Antibiotics Vomiting Bacteria Cephalosporin Rash Adverse drug reaction

D is not correct. 5°/o chose this.

6
lock Suspend
s 0
End Block
Item: 11 of 16 ~. , . M k <:] t> al ~· ~
QIO: 2008 .l. ar Previous Next lab 'lifllues Notes Calculator

2
C is not correct. 9°/o chose this.
3
Cephalosporins are ~-lactam antibiotics used to t reat infections caused by gram-positive and gram-negative
bacteria . The side effects associated with cephalosporins are relatively non-specific and may include nausea,
4
vomiting, diarrhea and a hypersensitivity rash. Use of cephalospor ins does not cause osteonecrosis.
5 Gram-negative bacteria Gram-positive bacteria Diarrhea Hypersensitivity Nausea Antibiotics Vomiting Bacteria Cephalosporin Rash Adverse drug reaction

6
D is not correct. 5°/o chose this.
7
Osmotic diuretics such as mann itol are used to treat shock and drug overdose. Common side effects seen with
•8 osmotic diuretic use are headaches, dizziness, nausea, and vomiting . Osm otic diuretics are not associated with
9 osteonecrosis .
Diuretic Mannitol Nausea Vomiting Osmosis Dizziness Drug overdose Osmotic diuretic
10
11 E is not correct. 19% chose this.
• 12 The vinca alkaloids, such as vincristine and vinblastine, are microtubule inh ibitors used in the treatment of
testicu lar ca rcinoma and Hodg kin and non- Hodg kin lymphomas. One of the most well known and high yield side
• 13
effects associated with these drugs is peripheral neu ropathy. Vinca al kaloids are not associated with
• 14 osteonecrosis .
• 15 Vinblastine Microtubule Vincristine Peripheral neuropathy Vinca alkaloid Alkaloid Testicular cancer lymphoma Carcinoma Adverse drug reaction Vinca

• 16

Bottom Line:
Bisphosphonates, which are used to treat metastatic bone disease, lytic lesions of multiple myeloma, and
osteoporosis, can cause osteonecrosis of the jaw.
Osteoporosis Multiple myeloma Bone metastasis Bisphosphonate Avascular necrosis Osteonecrosis of the jaw Metastasis Bone disease lytic cycle lysis Bone

6
lock
s
Suspend
0
End Block
Item: 11 of 16 ~ 1 • M k -<:J 1>- Jil ~· !:';-~
QIO: 2008 ..L ar Pre vious Next Lab fli!ltues Not es Calculat o r

1 •

2 FA17 p 4 58.5
3 Bisphosphonates Alendronate, ibandronate, risedronate, zoledronate.
4 MECHANISM Pyrophosphate analogs; bind hydro,yapatite in bone, inhibiting osteoclast acti,·ity.
5 CLINICAl USE Osteoporosis, hypercalcemia, Paget disease of bone, metastatic bone disease, osteogenesis
6 imperfecta.
7 ADVERSE EFFECTS Esophagitis (if taken ora lly, patients are ad' ised to take with water and remain upright for 30
·8 minutes), osteonecrosis of ja\\, at} pica I stress fractures.
9
10 FA17 p 409.1

11 Multiple myeloma 1\ lonoclonal plasma cell ("fried egg'' Think CR.\B:


• 12
appearance) cancer that arises in the marrow llyperCalcemia
Mspike-'\ and produces large amounts of lgC (55%) or Renal involvement
• 13
Ig (25%). Bone marrow > I0% monoclonal Anemia
• 14 plasma cells. lost common 1° tumor arising Bone lytic lesions/Back pain
• 15 within bone in people> 40-50 yea rs old. ~l ultiple Myeloma: V1onoclonal M protein
• 16 Associated with: spike
t susceptibility to infection Distinguish from Waldenstrom
• Primary amyloidosis (AL) macroglobulinemia .... ,\!J. spike= lgM
• Punched-out lytic bone lesions on x-ray tJ .... hypen·iscositr syndrome (eg, blurred vision,
.\t1 spike on serum protein elect rophorcsis Raynaud phenomenon); no "CRAB" findings .
lg light chains in urine (Bence Jones
protein)

a
Lock
s
Suspend
8
End Bl ock
Item: 11 of 16 ~ 1 • M k -<:J 1>- Jil ~· !:';-~
QIO: 2008 ..L ar Pre v ious Next Lab fli!ltues Not es Calcula t o r

1 •
FA17 p 435.2
2
Osteoporosis Trabecular {spongy) and cort ica I bone lose mass Can lead to vertebral compression fractures (fJ,
3
Normal Normal and interconnections despite normal bone small arrows; large arrows show normal-for-age
4 ~ntel\'ellebral ontetVenebral mineralization and lab ,·a lues (serum Ca 2+ and ,·ertebral body height for comparison)-acute
5 do~ r~:~•mm P0_.3-). back pain, loss of height, kyphosis. Iso can
6 lost commonly due to t bone resorption present with fractures of femoral neck, distal
related to l estrogen b ·els and old age. radius (Calles fracture).
~
7
Can be secondary to drugs (eg, steroids,
·8
alcohol, anticom ulsants, anticoagulant ~.
9

i:
thyroid replacement therapy) or other
Central expaOSIOrl RestrlCted
10 of intetVenebral ontetVenebral medical conditions (eg, hyperparathyroidism,
do~ foramen
11 .v hyperthyroid ism, multi pie myeloma,
• 12 ·~ ~ ~ malabsorption syndromes).
Diagnosed by a bone minera l density scan (dual-
• 13 ~>-. ~ energy x-ray absorptiomctry fOF.XA]) with a
• 14
old
M compression friKture T-score of::;; -2.5 or by a fragility frtt cturc of
• 15 hip or ,·ertebra. Screening recommended in
• 16 women > 65 years old.
Prophylaxis: regular weight-bearing exercise
and adequate Ca 2+ and vitamin D intake
throughout adulthood.
Treatment: bisphosphonates, teriparat ide,
SER.\IIs, rarely calcitonin; denosumab
(monoclonal antibody against RA NKL).

a
Lock
s
Suspend
8
End Bl ock
Item: 12 of 16 ~ 1 • M k -<:J 1>- Jil ~· !:';-~
QIO: 4572 ..L ar Pre v ious Next Lab fli!ltues Notes Calcula t o r

1 •
A 29-year-old man with a 3-year history of hip and lower back pain compla ins of morning stiffness of his
2
back that improves w ith exercise. His pain was initially controlled w ith NSAID use, but the patient is no
3 longer find ing rel ief with this treatment. He is otherwise healthy. His physical examination is unremarkable
4 except for a decrease in chest expansion . X-ray of the spi ne is shown in the image.
5
6
7
·8
9
10
11

• 12
• 13
• 14
• 15
• 16

a
Lock
s
Suspend
8
End Bl ock
2

3
4
5
6
7
·8
9
10

11

• 12
• 13
• 14
• 15
• 16

a
Lock
s
Suspend
8
End Block
2

3
4
5
6
7
·8
9
10

11
The physician suggests an adjunctive medication to help control the patient's symptoms . Howeve r, to prevent
• 12
potential serious adverse effects associated w ith the alternative treatment, which test should be adm inistered first?
• 13
:
• 14
A. Blood pressure
• 15
• 16
B. Erythrocyte sedimentation rate
C. Neisseria gonorrhoeae nucleic acid hybrid ization test
D. Pu lmona ry function test
E. Purified protein derivative skin test

a
Lock
s
Suspend
8
End Block
Item: 12 of 16 ~. I • M k <:] t> al ~· ~
QIO: 4572 .l. ar Previous Next lab 'lifllues Notes Calculator

2 The correct answer is E. 51 °/o chose this.


3 This patient has ankylosing spondyl itis; the image shows the "bamboo spine" characteristic of this disease. An
4
effective t reatment is infliximab, an antitumor necrosis factor -a monoclonal antibody that relieves symptoms
and slows the prog ression of the disease . However, due to its immunosuppressive properties, a rare but serious
5 adverse eff ect is opportunistic infection, especial ly flare-up of dor mant tube rculosis. A purified protein derivative
6 (PPD) skin test shou ld be conducted prior to t reatment. I f resu lts are positive, the patient should be sent for an
7 X-ray of the chest and treated wit h antitubercu lar medications prio r to initiating treatment with infliximab.
Opportunistic infection Infliximab Ankylosing spondylitis Tuberculosis Chest radiograph Monoclonal antibody Immunosuppression Protein Spondylitis Antibody
•8
Necrosis Mantoux test X-ray Mycobacterium Adverse effect Management of cancer
9
10 A is not correct. 8°/o chose this.
11 I ncrease or decrease of blood pressu re may occur afte r infusion of infliximab, but is a common and temporary
reaction that usual ly resolves quickly.
12
Infliximab Blood pressure Infusion
• 13
• 14
B is not correct. 13% chose this .
• 15
A high erythrocyte sed imentation rate is a nonspecific find ing in inflammatory processes and is seen in other
processes as wel l (ie, ma lignancy, aging) . Although the eryth rocyte sedimentation rate may be elevated in a
• 16 patient with an ky losing spondyl it is, it does not confirm the diagnosis and does not indicate potential adverse
effects .
Ankylosing spondylitis Erythrocyte sedimentation rate Red blood cell Cancer Malignancy Inflammation

C is not correct. 7°/o chose this.


Many patients with Neisseria gonorrhoeae infection exper ience arth ritis- like jo int pain, most common ly in the
knees. Th is patient's long histo ry of back pain does not suggest gonorrhea infection. In addition, infl iximab
~ .. --~---+- . . . . . . . .- --+- :. . . . . . . . . . . .-.. . +-h- .... :.- 1... -+ __ ..............h ......... l ; .... +.............. ;........

6
lock
s
Suspend
0
End Block
Item: 12 of 16 ~. I • M k <:] t> al ~· ~
QIO: 4572 .l. ar Previous Next lab 'lifllues Notes Calculator

1 processes as wel l (ie, malignancy, aging) . Although the erythrocyte sedimentation rate may be elevated in a
2
patient with ankylosing spondyl itis, it does not confirm the diagnosis and does not indicate potential adverse
effects .
3 Ankylosing spondylitis Erythrocyte sedimentation rate Red blood cell Cancer Malignancy Inflammation
4
C is not correct. 7°/o chose this.
5
Many patients with Neisseria gonorrhoeae infection experience arthritis- like joint pain, most common ly in the
6 knees. Th is patient's long history of back pain does not suggest gonorrhea infection. In addition, infl iximab
7 treatment does not increase the risk of gonorrhea l infection.
Infliximab Gonorrhea Neisseria gonorrhoeae Arthralgia Neisseria
•8

9 Dis not correct. 21% chose this.


10 The use of infliximab is associated with an increased incidence of pneumonia . Pulmonary function tests,
11 however, would not be predictive of the like lihood of this adverse effect. I nfliximab does not otherwise
commonly affect lung function .
12
Infliximab Pneumonia Spirometry Pulmonary function testing lung Adverse effect
• 13
• 14
• 15 Bottom Line:
• 16 Ankylosing spondyl itis is treated with the monoclonal antibody infliximab. Due to its immunosuppressive
properties, rare serious opportunistic infections may occur, especia lly flare-ups of dormant tubercu losis. A
positive PPD skin test indicates that the patient shou ld be sent for an X-ray of the chest and treated with
antitubercu la r medications prior to initiating treatment with infliximab.
Infliximab Ankylosing spondylitis Tuberculosis Chest radiograph Monoclonal antibody Spondylitis Immunosuppression Antibody Opportunistic infection
Mycobacterium X-ray

6
lock
s
Suspend
0
End Block
Item: 12 of 16 ~ 1 • M k -<:J 1>- Jil ~· !:';-~
QIO: 4572 ..L ar Pre v ious Next Lab fli!ltues Notes Calcula t o r

1 •

2
FA17 p 442.1

3 Seronegative Arthritis without rheumatoid factor (no anti-lgC antibody). Strong association with HLA-8 27
4
spondyloarthritis (l\ IHC class I serot) pe). Subtypes (P \IR) share ,·ariable occurrence of inflammatory back
pain (associated with morning stiffness, impro,·es "ith exercise), peripheral arthritis, enthesitis
5
(inflamed insertion sites of tendons, eg, Achilles), dactylitis (..sausage fingers"), uveitis.
6
Psoriatic arthritis Associated with skin psoriasis and nail lesions. Seen in fe"er than 11> of patients " ith psoriasis.
7 symmetric and patchy im·olvement
·8 Dactylitis and "pencil-in-cup" deformity of
9 DIP on x-rav• g .
10 Ankylosing Srmmctric im·olvemcnt of spine and sacroil iac Bamboo spine (,·ertebral fusion) . Can cause
11 spondylitis joints - ankylosis (joint fusion), uveitis, aortic restrictive lung disease due to limited chest
12
regurgitation. wall expansion (costovertebral and costostcrnal
ankylosis).
• 13
lore common in males .
• 14
Inflammatory bowel Crohn disease and ulcerative colitis arc often
• 15
disease associated with spondyloarth ritis.
• 16
Reactive arthritis Formerly known as Reiter syndrome. "Can' t see, can' t pee, can't bend my knee."
Classic triad: Shigella, Yersinia, Chlamydia, Campylobacler,
Conjuncti\ iti~ Sahnonella (ShY ChiCS).
• Urcth ritis
\ rthritis

a
Lock
s
Suspend
8
End Bl ock
10

11 FA17 p 457.3

12 TN F-a inhibitors All T 1F-a inhibitors predispose to infection, including reactivation of latent TB, since T lF is
• 13 important in granuloma formation and stabilization .
DRUG MECHANISM CLINICALUSE
• 14
Etanercept Fusion protein (receptor for TN F-a + lgC1 Fe), Rheumatoid arthritis, psoriasis, ankylosing
• 15
produced by recombinant D A. spondylitis
• 16
Etanercept intercepts T Nl'.
lnfliximab, Anti-T F-a monoclonal antibody. lnAammatory bowel disease, rheumatoid arthritis,
adalimumab, ankylosing spondylitis, psoriasis
certolizumab,
golimumab

a
Lock
s
Suspend
8
End Block
Item: 13 of 16 ~. I • M k <:] t> al ~· ~
QIO: 3479 .l. ar Previous Next lab 'lifllues Notes Calculator

1
A 23-year-old man presents to his physician after losing consciousness. A friend who was with the man at ~~AI
2
the t ime told him that, whi le he was unconscious, he loo ked like he was having a "fit." The friend reports that
3 the patient lost control of bowel and bladder function during the episode . The patient denies drug or alcoho l
4 use. The physician prescribes him a med ication to contro l the problem . Three weeks later, the patient returns to
the doctor to report a rash that has deve loped since he started taking the medication . The rash invo lves both the
5
skin and the mucous membranes. The rash began as macular and then deve loped bu llae that ruptured, as shown
6 in the image . Despite stopping the drugs, the patient dies .
~~--------------~----------~
7
•8

9
10
11

12
• 13
• 14
• 15
• 16

Image copyright © J Oral Maxillofac Pathol

6
lock
s
Suspend
0
End Block
Image copyright © J Oral Maxillofac Pathol
11
12
Which of the following medications most likely caused this rash?
• 13
:
• 14
A. Clonazepam
• 15
• 16
B. Gabapentin
C. Lamotrigine
D. Phenobarbita l
E. Topiram ate

a
Lock
s
Suspend
8
End Block
Item: 13 of 16 ~. I • M k <:] t> al ~· ~
QIO: 3479 .l. ar Previous Next lab 'lifllues Notes Calculator

2 The correct answer is C. 51°/o chose this.


3 The patient presented with a seizure disorde r. Specifica lly, his symptoms are those of having had a tonic-clonic,
4
or grand mal, seizu re. Among the drugs used to treat th is disorde r is lamotrigine. Although its exact mechan ism
of action is unknown, it is hypothesized that th is dr ug acts on voltage-sensit ive sod ium channels. The most
5
ser ious adverse reaction to lamotrigine is Stevens-Johnson syndrome, which is an immune-co mplex-med iated
6 hypersensitivity reaction that involves the mucocutaneous surfaces and that is potential ly fatal. I t is important
7 to discontinue lamotrigine in patients who deve lop a rash from ta king lamotrigine. Stevens-Johnson synd rome is
•8
also associated with the use of the anticonvu lsants phenytoin, ethosuximide, and ca rbamazepine .
Ethosuximide Stevens-Johnson syndrome Carbamazepine lamotrigine Phenytoin Anticonvulsant Sodium channel Epilepsy Hypersensitivity Seizure types
9
Adverse drug reaction Adverse effect Generalised tonic-clonic seizure Sodium Mechanism of action Rash Epileptic seizure
10
11
A is not correct. 11% chose this.
Clonazepam is a benzodiazepine that acts as an anxio lytic and an anticonvulsant. It increases the frequency of
12
y-aminobutyric acid channels opening, and it is used to t reat absence and myoclonic seizu res. The patient in
13 this question appea rs to have had a tonic-clonic seizure, so its use is not indicated. I ts adverse effects incl ude
• 14 ataxia, dizziness, and sedation .
Benzodiazepine Anxiolytic Anticonvulsant Clonazepam Ataxia Generalised tonic-clonic seizure Sedation Myoclonus Dizziness Myoclonic epilepsy Epileptic seizure
• 15
Seizure types
• 16

B is not correct. 11% chose this.


Gabapent in is an anticonvulsant that is used as part of a combination of drugs to treat seizu res and neu ropathic
pain. Stevens-Johnson syndrome is a rare adverse effect of this drug; more common adve rse effects include
sedation, dizziness, ataxia, and fatigue.
Stevens-Johnson syndrome Anticonvulsant Gabapentin Neuropathic pain Ataxia Sedation Epileptic seizure Dizziness Fatigue (medical) Adverse effect

Perioheral neurooathv

6
lock
s
Suspend
0
End Block
Item: 13 of 16 ~. I • M k <:] t> al ~· ~
QIO: 3479 .l. ar Previous Next lab 'lifllues Notes Calculator

1 D is not correct. 18% chose this.


2 Phenobarbita l, a barbitu rate, is an effective antiepi leptic drug for both simp le and complex partial seizures, as
3 well as general ized tonic-clonic seizures. Phenobarbital works on y-aminobutyric acid receptors, increasing the
4 duration of chlo ride channel opening . Principal adverse effects include centra l nervous system depression,
sedation, dizziness, and nystagmus. It can also lead to the deve lopment of Stevens-Johnson syndrome (SJS).
5
6
Phenobarbita l is general ly much mo re sedating than other antiepileptic drugs, such as lamotrigine, and is
the refo re not frequently used in the outpatient setting . Phenoba rbita l is more often used as a treatment for
7
refractory seizu res or status epilepticus, and commonly sedates patients to a point where they require
•8 intubation. I t wou ld the refore be an un likely cause of SJS in this patient, who was being treated as an
9 outpatient with a less sedating medication.
10 Barbiturate Stevens-Johnson syndrome lamotrigine Phenobarbital Nystagmus Central nervous system Status epilepticus Chloride channel Anticonvulsant Sedation

11 Dizziness Seizure types Epileptic seizure Central nervous system depression Generalised tonic-clonic seizure Partial seizure Intubation Major depressive disorder

12 Nervous system Sedative Depression (mood) Pharmaceutical drug Chloride

13 E is not correct. 9°/o chose this .


• 14 Topiramate is an anticonvu lsant that is used for seizu res and for migraine prophylaxis. I t likely acts on sodium
• 15 channels in neurons. I ts adverse effects include psychomotor slowing, sedation, and rena l stones .
Anticonvulsant Topiramate Migraine Epileptic seizure Sedation Sodium channel Kidney stone Sodium Preventive healthcare Neuron Kidney
• 16

Bottom Line:
Lamotrig ine, an antiepileptic, is associated with Stevens-Johnson syndrome, which produces a seve re rash and
is potentially fatal.
Stevens-Johnson syndrome lamotrigine Anticonvulsant Rash

6
lock
s
Suspend
0
End Block
Item: 13 of 16 ~ 1 • M k -<:J 1>- Jil ~· !:';-~
QIO: 34 79 ..L ar Pre v ious Next Lab fli!ltues Not es Calcula t o r

1 •
FA17 p514.1
2
Epilepsy drugs
3 GENERALIZED
4 ~

.....
< ~ V'>
5 0
~
:z:
0
.....
~ ...:z..... .....
::::>

...... .,., ;::::


~
< v =>::h
6 ;:::: ,_ ~
:z: -
<0..
::""
V'>
,_
0
<
,_
V'> ... MECHANISM SIDE EFFECTS NOTES
7
Ethosuximide • Blocks thalamic T-t~-pe Ca~• El GI IIJ- Fthosm:imide Sucks to have Silent
·8 ./ channels causes Fatigue. Gl distress, (absence) Seizures
9
lleadache, Itching (and
urticaria). and Ievens-
10 Johnson syndrome
11 Benzodiazepines •• t C, \BA,, action Sedation, tolewnce, Also for eclampsia seimres (1st
(eg, diazepam, ./ dependence, respiratory line is l\lgS04)
12 lorazepam, depre~~ion
13 midazolam)
• 14 Phenobarbital ./ ./ t CAB/\, action Sedation, tolerance, Ist line in neonates
dependence, induction
• 15 of cytochrome P--150,
• 16
cardiorespiratory depression
Phenytoin, ./ • uo
Blocks a+ channels; 1ero- \Jcurologic: nystagmus, diplopia, ataxia, sedation, peripheral
fosphenytoin ./ ./ order kinetics neuropathy. Dcrmatologic: hirsutism, Ste1·ens-)ohnson
~) ndrome, gingival hyperplasia, DRESS syndrome.
l\lusculoskeletal: osteopeni:J, SLE-like syndrome. llematologic:
megaloblastic anemia. Reproductive: teratogenesis (fetal
hydantoin S) ndrome). Other: C)tochrome P-4;0 induction
Carbamazepine ./ Blocks a• channels Diplopia, ata:..ia, blood 1st line for trigeminal neuralgia
./ d1 ~r:tii:ti (:urranulocvto~is .

a
Lock
s
Suspend
8
End Bl ock
Item: 13 of 16 ~ 1 • M k -<:J 1>- Jil ~· !:';-~
QIO: 34 79 ..L ar Pre v ious Next Lab fli!ltues Not es Calcula t o r

1 •
Phenytoin, ./ • *** Blocks 1a+ channels; 1ero- \Jeurologic: nystagmus, diplopia, ataxia, sedation, peripheral
2 fosphenytoin ./ ./ order ki nctics neuropathy. Oermatologic: hirsutism, Stc\'ens-)ohnson
S) ndrome. gingival hyperplasia, DRESS syndrome.
3 ~ lnsculoskeletal: osleopenia, SLE-Iike syndrome. llematologic:
4 megalobl.istic anemia. Reproducti\'e: teratogenesis (fetal
hyda ntoin S) ndrome). Other: C)tochrome P-4;Qinduction
5
Carbamazepine ./ Blocks a+channels Diplopia, ala\ia, blood 1st line for trigeminal neuralgia
6 ./ d) scrasia} (agranulocytosis.
aplastic anemia), Ji,er
7
toJo.icit). teratogenesis.
·8 induction of C\ tochrome
P-450, SIADI i, Ste\ellS-
9
Johnson ~) ndrome
10 Valproic acid ./ • ./ t 'a+ channel inacti\'ation, C l distress. rare but fatal Also used for myoclonic seizures,
11 ./ t CABA concentration hepatotoxicity (measure bipolar disorder, migraine
by inhibiting CABA LFTs), pancreatitis, neural prophylaxis
12 transaminase h1be defects, tremor, weight
gain, contraindicated in
13
pregnancy
• 14
Vigabatrin t CABA by irrcvcrsibl) Permanent visual loss (black
• 15 inhibiting CABA bo" warning)
transaminase
• 16
Gabapentin Primarily inhibits high-voltage- Sedation, ataxia Also used for periphera I
activa ted Ca 2+ channels; neuropathy, postherpetic
designed as CABA analog neuralgia
Topiramate ./ ./ Blocks 'a+ channels, t C \ Br\ Sedation, mental dulling, Also used for migraine
action kidney ~I ones, weight loss, pre,·ention
glaucoma
lamotrigine ./ ./ ./ Blocks , oltage-gated la+ Ste\ens-Johnson S) ndrome
l"'h'llnnPic tnhil,i t~ ti-l~ ,.,..)~""' .... cP fnH•<t h~ titMt~¥1 <lnu h·\

a
Lock
s
Suspend
8
End Bl ock
Item: 13 of 16 ~ 1 • M k -<:J 1>- Jil ~· !:';-~
QIO: 34 79 ..L ar Pre v ious Next Lab fli!ltues Not es Calcula t o r

1 induction of C) tochrome •
P-450, SIAOII, Stelens-
2 Johnson syndrome
3 Valproic acid ./ • ./ I Na+ channel inactivation, Cl distress. rare but fatal Also used for myoclonic seizures,
4
./ I CA BA conccntrntion hepatotoxicity (measure bipolar disord~r, migraine
by inhibiting CABA LFTs), pancreatitis, neural prophylaxis
5 trn nsa m inase tube defects, tremor, weight
gain, contraindicated in
6
pregnancy
7 Vigabatrin I C.\ BA b} irrcl'crsibl) Perm.ment 'isualloss (black
·8 inhibiting C.\ 8.\ bo' 11arning)
t rnnsa m inase
9
Gabapentin ./ Primaril} inhi~its high-\ol tage- Sedation. ata,ia Also used for periphernl
10 actil'ated Ca-• channels; neuropathy, postherpetic
designed as CABA analog neuralgia
11
Topiramate ./ ./ Blocks 'a+ channels, I C \ 1:3/\ Sedation, mental dulling. Also used for migraine
12 action kidney ~ tones, weight loss, prevention
13 glaucoma
• 14 Lamotrigine ./ ./ ./ Blocks voltage-gated 1 a 1 Stevens-Johnson syndrome
channels, inhibits the release (must be tit rn ted slowly)
• 15 of glutamate
• 16 Levetiracetam ./ ./ Unknown; may modulate I•:1t igue, drows iness,
CABA and glutamate release hcad<~che, neuropsychiatric
svmptoms (eg, personality
changes)
Tiagabine ./ I CABA by inhibit ing rcuptakc

= 1st line; = lst line for acute; ••• = lst line for prophylaxis.

a
Lock
s
Suspend
8
End Bl ock
Item: 14 of 16 ~. I • M k <:] t> al ~· ~
QIO: 2125 .l. ar Previous Next lab 'lifllues Notes Calculator

1
A previously hea lthy 27- yea r-old pregnant woman visits her doct or for a routine physica l check-up and is ~~AI
2
found t o have a blood pressu re of 160/90 mm Hg . Her doct or prescr ibes an antihypertensive med icat ion. One
3 week later the patient returns to the office complaining of fever, fatigue, and diffuse muscle aches and j oint
4 pain . Physica l exam inat ion is not able for a sca ling, erythemat ous rash on the woman's cheeks and t he bridge of
her nose, as well as on her forearms. Laboratory tests revea l elevat ed blood urea nitrogen and creatinine levels.
5
6
Wh ich of the fo llowing best describes the mechanism of action of the drug t hat t his patient was mostly likely
7
prescribed?
•8
:
9
A. Blocks ~ - ad renerg i c receptors
10
B. Blocks ang iotensin II receptors
11

12 C. Causes vasod ilat ion of arterioles


13 D. Inhibits angiotensin- converting enzyme
• 14
E. Inhibits resorption of chlor ide and sodium in dista l convo luted tubu le
• 15
• 16

6
lock
s
Suspend
0
End Block
Item: 14 of 16 ~ 1 • M k -<:J 1>- Jil ~· !:';-~
QIO: 2125 ..L ar Prev ious Next Labfli!ll ues Notes Calculat o r

1 •

2 The correct answer is C. 52°/o chose this.


3 The onset of constitutional symptoms, arthralgia/myalgia, malar rash (shown
4
in the image), and kidney dysfunct ion 1 week after the start of a new
medication ra ises concerns about drug-induced lupus. This patient was likely
5
given hydralazine, a peripheral vasodilator used as an antihypertensive,
6 especially during pregnancy. Neurologic and renal disease are not common in
7 drug-induced lupus, though they can occur. However, hydralazine in
particular can cause an ANCA- positive vasculitis that damages the kidneys. In
·8
addition, almost all patients will t est positive for anti-histone antibodies.
9 Treatment consists of discontinuing the drug; corticosteroids are usually not
10 necessary. In addition to Hydralazine, other drugs that cause lupus-like
11
syndrome are: Sulfa-drugs, I NH, Procainamide, Phenytoin, Etanercept.
(Remember, Having lupus is "SHIPP-E .")
12 Hydralazine Vasodilation Anti-histone antibodies Malar rash Antihypertensive drug vasculitis Corticosteroid
13
Systemic lupus erythematosus Lupus erythematosus Antibody Kidney disease Kidney Pregnancy Rash
14
Pharmaceutical drug
Image courtesy of Wikimedia
• 15 Commons
• 16
A is not correct. 8°/o chose this.
~-B l ockers are used to t reat hypertension and are generally safe during pregnancy . However, they are not
commonly associated with drug-induced lupus.
Hypertension Lupus erythematosus Systemic lupus erythematosus Pregnancy

B is not correct. 9 °/o chose this.


Angiotensin II receptor blockers such as losartan are used to treat hypertension but are contraindicated during

a
Lock
s
Suspend
8
End Bl ock
Item: 14 of 16 ~. I • M k <:] t> al ~· ~
QIO: 2125 .l. ar Previous Next lab 'lifllues Notes Calculator

1
B is not correct. 9°/o chose this .
2 Angiotensin II receptor blockers such as losartan are used to treat hypertension but are contra ind icated during
3 pregnancy . These drugs can cause adverse effects simi lar to those of ACE inh ibitors, although genera lly they are
4 well t olerated.
losartan Angiotensin Angiotensin II Hypertension ACE inhibitor Angiotensin II receptor antagonist Contraindication Receptor (biochemistry) Pregnancy
5
6 D is not correct. 16% c hose this.
7 Angiot ensin- converting enzyme (ACE) inh ibitors such as captopril and enalapri l are used to treat hypertension
but are contra ind icat ed during pregnancy . Although ACE inhibitors have been associated with drug -induced
•8
lupus in case reports, t hey are not a common cause of t his syndrome. ACE inh ibitors can, however, cause
9 hyperkalemia, cough, acute renal fai lure, and, rarely, ang ioedema .
10 Enalapril Captopril Hyperkalemia ACE inhibitor Angiotensin-converting enzyme Angioedema Enzyme Acute kidney injury Hypertension Contraindication Cough

11 Pregnancy Systemic lupus erythematosus Kidney

12 E is not correct. 15% c hose this.


13 Th iazide diuretics such as hydroch lorothiazide often are used t o treat hypertension but are not recommended
14 during pregnancy because of their abi lit y to cross the placent al barrier and cause thrombocyt openia and
• 15 electrolyt e imbalances in the fetus . There is no definitive evidence yet to show that thiazides can cause drug -
induced lupus, alt hough there are isolated reports of this association . Thiazides cause many met abolic
• 16
abnormal it ies (eg, hypokalemia, hyponatremia, hyperg lycemia, hyperlipidem ia, hypercalcemia).
Hydrochlorothiazide Hyponatremia Hypokalemia Thiazide Hypercalcaemia Thrombocytopenia Hyperglycemia Hyperlipidemia Electrolyte Hypertension Diuretic

Placenta Pregnancy Fetus Systemic lupus erythematosus Metabolism

Bottom Line:
~-- · -- ""''- -""' --- -- - --=-""- -1 ...:. . 1- - · · · - · · - 1: 1 .- - -··- -1 ---- --- llf""' • • I.C- -1 .... . - - 11 •• -1 .. - 1- - : - - "1" 1\ 11 1 ft - - - - : - - - - : -1 -

6
lock Suspend
s 0
End Block
Item: 14 of 16 ~. I • M k <:] t> al ~· ~
QIO: 2125 .l. ar Previous Next lab 'lifllues Notes Calculator

1 well tolerated.
losartan Angiotensin Angiotensin II Hypertension ACE inhibitor Angiotensin II receptor antagonist Contraindication Receptor (biochemistry) Pregnancy
2

3 D is not correct. 16% chose this.


4 Angiotensin-converting enzyme (ACE) inh ibitors such as captopril and enalapr il are used to t reat hypertension
5 but are contraind icated during pregnancy . Although ACE inhibitors have been associated with drug-induced
lupus in case reports, they are not a common cause of th is syndrome. ACE inhibitors can, however, cause
6
hyperkalemia, cough, acute rena l fai lure, and, rarely, ang ioedema .
7 Enalapril Captopril Hyperkalemia ACE inhibitor Angiotensin-converting enzyme Angioedema Enzyme Acute kidney injury Hypertension Contraindication Cough
•8 Pregnancy Systemic lupus erythematosus Kidney
9
E is not correct. 15% chose this.
10
Thiazide diuretics such as hydroch lorothiazide often are used to treat hypertension but are not recommended
11 during pregnancy because of their abi lity to cross the placental barrier and cause thrombocytopen ia and
12 electrolyte imba lances in the fetus . There is no definit ive evidence yet to show that thiazides can cause drug-
13 induced lupus, although there are isolated reports of this association . Th iazides cause many metabol ic
abnorma lities (eg, hypokalemia, hyponatremia, hyperg lycemia, hyperl ipidemia, hypercalcemia) .
14
Hydrochlorothiazide Hyponatremia Hypokalemia Thiazide Hypercalcaemia Thrombocytopenia Hyperglycemia Hyperlipidemia Electrolyte Hypertension Diuretic
• 15
Placenta Pregnancy Fetus Systemic lupus erythematosus Metabolism
• 16

Bottom Line:
Drugs that are associated with a lupus-like syndrome are Su lfa-d r ugs, Hydralazine, INH, Procainamide,
Phenytoin, Enterecept. (Having lupus is "SHIPP-E ")
lupus erythematosus Systemic lupus erythematosus

6
lock
s
Suspend
0
End Block
Item: 14 of 16 ~ 1 • M k -<:J 1>- Jil ~· !:';-~
QIO: 2125 ..L ar Pre v ious Next Labfli!llues Notes Calcula t o r

1 •

2 FA17 p241.2

3
Drug reactions- musculoskeletal/skin/connective tissue
DRUG REACTION CAUSAL AGENTS NOTES
4
Fat redistribution P rotease inhibitors, G lucocorticoids r at P iG
5
Gingival hyperplasia Phenytoin, Ca 2'" channel blockers, cyclosporine
6
Hyperuricemia (gout) Pvrazinamide, T hia1ides, Furosemide, "\ iacin, Painful Tophi and Feet eed C are
7 '
C~closporine
·8
Myopathy Statlns, fibrates, niacin, colchicine, daptomycin,
9
hydrox,·chloroquine, interferon-a,
10
penicillamine, glucocorticoids
11
Osteoporosis Corticosteroids, depot medroxyprogesterone
12 acetate, GnR!-1 agonists, aromatase inhibitors,
13 anticonvulsants, heparin
14 Photosensitivity Sulfonamidcs, Amiodaronc, Tetracyclines, 5-F U SAT For Photo
• 15 Rash (Stevens- Anti-epileptic drugs (especially lamotrigine), Stc,cn Johmon has epileptic allergy to mlfa
• 16 Johnson syndrome) allopurinol, sulfa drugs, penicillin drugs and penicillin
SLE-Iike syndrome Sulfa d rugs, Hydralazine, Isoniazid, I laving lupus is "SIIIPP-E "
P rocainamide, Phenytoin, Etancrccpt
Teeth discoloration Tctracycl ines Tceth racyclincs
Tendonitis, tendon Pluoroquinolones
rupture, and
.. '
a
Lock
s
Suspend
8
End Bl ock
Item: 14 of 16 ~ 1 • M k -<:J 1>- Jil ~· !:';-~
QIO: 2125 ..L ar Pre v ious Next Labfli!llues Notes Calcula t o r

1
Osteoporosis
-
Corticosteroids, depot medroxyprogesteronc

2
acetate, GnRH agonisls, aromatase inhibitors,
3 anticonvulsants, heparin
4 Pho to sensitivity Sulfonamides, Amiodaronc, Tetracyclines, 5-F U S.\T For Photo
5 Rash (St evens- Anti-epileptic drugs (especially lamotrigine), Stc\ cn Joh mon has epileptic allergy to ~ulfa
6 Johnson syndrome) allopurinol, sulfa drugs, penicillin drugs and pen icillin
7 SLE-Iike syndrome Sulfa drugs, Il)dralazine, Isoniazid, lla\'ing lupus is "SI liPP-E"
·8 P rocainamide, P henytoin, Etanc rcept
9 Teeth discoloration Tctracvcl i nes Tccth racyclines
10 Tendonitis, tendon Fluoroquinolones
11 rupture, and
12 cartilage damage

13
14 FA17 p 304.3
• 15 Hydralazine
• 16 MECHANISM t cGM P .... smooth muscle rclaxa t ion. asodi latcs arterioles> veins; a Fterload reduct ion .
CLINICAL USE Severe hypertension (particularly acute), I I Ji' (with organic nitrate). Safe to use during pregnancy.
Frequently coadministcrcd with a ~-blocker to prc\·cnt rcAcx tachyca rdia.
ADVERSE EFFECTS Compensatory tachycardia (contraindicated in angina/CAD), Auid retention, headache, angina.
Lupus-1 ike syndrome.

a
Lock
s
Suspend
8
End Bl ock
Item: 15 of 16 ~ 1 • M k -<:J 1>- Jil ~· !:';-~
QIO: 4710 ..L ar Pre v ious Next Lab fli!ltues Not es Calcula t o r

2
A 60-year-old man with a history of alcohol abuse presents to a rheumatology cl inic for eva luation of severe, IAA] •
recurrent "gnawing" pa in in his f ingers and toes. On examination he has large tophi at his elbows. He is
3 diagnosed with chron ic gouty arthritis, and in addition to being advised to quit drinking, he is given a dr ug
4 that competes w ith uric acid for reabsorption in the kidney.
5
6 What med ication has most likely been prescribed?
7 :

·8 A. Allopurinol
9 B. Colchicine
10
C. Furosemide
11
D. I ndo methacin
12
13 E. Low-dose asp irin
14 F. Probenecid
0 15
0
16

a
Lock
s
Suspend
8
End Bl ock
Item: 15 of 16 ~. I • M k <:] t> al ~· ~
QIO: 4710 .l. ar Previous Next lab 'lifllues Notes Calculator

1
The correct answer is F. 61°/o chose this.
2 Probenecid and sulfinpyrazone are used to treat chronic gout. Both dr ugs act on the kidney to inh ibit uric acid
3 reabsorption in the proximal convoluted tubule.
Probenecid Sulfinpyrazone Uric acid Proximal convoluted tubule Gout Kidney
4
5 A is not correct. 13% chose this.
6 Al lopurinol blocks the enzyme xanthine oxidase, preventing the formation of uric acid from purines. I t also
7 increases the reuti lization of hypoxanthine and xanthine for nucleotide and nucleic acid synthesis. The resu ltant
increase in nucleotide concentration leads to feedback inhibition of de novo pu r ine synthesis. Overal l, this
•8
resu lts in decreased ur ic acid concentrations in serum and urine. Allopu r inol is the dr ug of choice for treating
9 chronic gout. In this question, the patient is presenting with an acute gout attack in the setting of chronic gout.
10 Al lopurinol is not t he t reatment of choice for acute gout exacerbations.
Xanthine oxidase Allopurinol Xanthine Uric acid Purine Hypoxanthine Gout Enzyme Nucleic acid Urine Enzyme inhibitor Blood plasma Nucleotide
11
De novo synthesis Mutation Oxidase Serum (blood)
12
13 B is not correct. 11% chose this.
14 Colch icine depolymerizes microtubu les, limiting the infla m matory response to urate crystals. Colchicine is used
15
to treat acute gout and can be used in prophylaxis as well.
Colchicine Gout Uric acid Inflammation Microtubule
• 16
C is not correct. 5°/o chose this.
Fu rosemide, a su lfona mide loop diuretic, actua lly raises ser um uric acid levels. It is specifica lly contraind icated
in individua ls with gout.
loop diuretic Furosemide Uric acid Diuretic Sulfonamide (medicine) Sulfonamide Gout Blood plasma Serum (blood) Contraindication

D is not correct. 5°/o chose this.

6
lock
s
Suspend
0
End Block
Item: 15 of 16 ~. I • M k <:] t> al ~· ~
QIO: 4710 .l. ar Previous Next lab 'lifllues Notes Calculator

1
B is not correct. 11% chose this.
2 Colch icine depolymerizes microtubu les, limiting the inflammatory response to urate crystals. Colchicine is used
3 to treat acute gout and can be used in prophylaxis as well.
Colchicine Gout Uric acid Inflammation Microtubule
4
5 C is not correct. 5°/o chose this.
6 Fu rosemide, a su lfonamide loop diuretic, actua lly raises ser um uric acid levels. It is specifica lly contraind icated
7 in individuals with gout.
loop diuretic Furosemide Uric acid Diuretic Sulfonamide (medicine) Sulfonamide Gout Blood plasma Serum (blood) Contraindication
•8
9 D is not correct. 5°/o chose this.
10 I ndomethacin, a nonsteroida l anti-inflammatory drug, limits the inflammatory response by blocking
prostag landin synthesis. It is preferable to colchicine in the treatment of acute gout.
11
Nonsteroidal anti-inflammatory drug Colchicine Indometacin Prostaglandin Gout Anti-inflammatory Inflammation
12
E is not correct. 5°/o chose this.
13
At high doses (>3 g/day), sa licylates prevent tubu lar resorption of uric acid. At low doses, however, sa licylates
14
compete with uric acid for excretion . Therefore, aspirin shou ld be used with caution in individua ls with gout.
15 Uric acid Aspirin Salicylic acid Gout Excretion

• 16

Bottom Line:
Probenecid and sulfinpyrazone inhibit reabsorption of uric acid in the proximal convoluted tubule.
Probenecid Uric acid Proximal convoluted tubule Sulfinpyrazone

6
lock
s
Suspend
0
End Block
Item: 15 of 16 ~ 1 • M k -<:J 1>- Jil ~· !:';-~
QIO: 4710 ..L ar Pre v ious Next Lab fli!ltues Not es Calcula t o r

1 •
FA17 p457.2
2 Gout drugs
3 Chron1c gout drugs (preventive)
4 Allopurinol Competiti,·e inhibitor of xanthine oxidase. Dtet - - Purines - - Nucleic acids
5 l conversion of hypoxanthine and xanthine to
6 urate. Also used in lymphoma and leukemia
Hypoxanthine
7
to pre,·ent tumor I}sis-associated urate
xanthine
nephropathy. t concentrations of azathioprine
·8
and 6-:\ IP (both normally metabolized by xanthl~xidase ) Allopurinol
9 xanthine oxidase). - febuxostat
Xanthine
10 ox1dase
Febuxostat Inhibits xanthine oxidase.
11 Plasma - - Urate crystals - Gout
Pegloticase Recombinant uricase that catalyzes metabolism uric acid deposited
12 of uric acid to allantoin (a more \\'ater-soluble in joints
13 product).
14 Probenecid Inhibits reabsorption of uric acid in proxim<d Tubular
convoluted tubule (also inhibits secret ion of
~
15 reabsorption

• 16 penicillin). Can precipitate uric acid calculi. Probenecid and


high-dose salicylates
Acute gout drugs ~f----==----- Tubular
NSAIDs Anr full-dose SAID (eg, naproxcn, ~ secretion

indomethacin). Avoid salicylatcs (may decrease Diuretics and


Urine low-dose salicylates
uric acid excretion, particularly at low closes).
Glucocorticoids OraJ, intra-articular, or parenteral.
- •• 0 0 . .. ' 1 • 1 ·•· • 1 1• • • I · I •.

a
Lock
s
Suspend
8
End Bl ock
urate. Also used in lymphoma and leukemia
2 Hypoxanthine
to pre\'ent tumor lysis- associated urate
3
4
nephropathy. t concentrations of azathioprine
and 6-:\IP (both normally mctaboli7cd by
l Xanthine
oxidase )
Xanthine - Allopurinol.
5 xanthine oxidase). - febuxostat
Xanthine
Febuxostat Inhibits xanthine oxidase. O)C]dase
6
7 Pegloticase Recombinant uricase that catalyzes metabolism Plasma - - Urate crystals - - Gout
uric add deposited
·8 of uric acid to allantoin (a more "ater-soluble in joints
9 product).
10 Probenecid Inhibits reabsorption of uric acid in proximal Tubular
conmluted tubule (also inhibits secretion of
~
reabsorptiOn
11
penicillin). Can precipitate uric acid calculi. Probenecid and
12 high·dose salicylates
Acute gout drugs 4-41--- ---=,...----- Tubular
13
~ secretion
NSAIDs Any full-dose SAID (eg, naproxcn,
14
indomethacin). Avoid sal icylates (may decrease Diuretics and
15 Urine low-dose salicylates
uric acid excretion, particularly at low doses).
• 16
Glucocorticoids Oral, intra-articular, or parenteral.
Colchicine Binds and stabilizes tubulin to inhibit
microtubule polymerization, impairing
neutrophil chemotaxis and degranulation.
Acute and prophylactic value. C l side effects.

a
Lock
s
Suspend
8
End Block
Item: 16 of 16 ~. I • M k <:] t> al ~· ~
QIO: 2476 .l. ar Previous Next lab 'lifllues Notes Calculator

1
A 19-year-old construction worker in Southern Ca lifornia presents to his primary care physician with a 6-
2
month history of acne vu lga r is that has on ly mi ldly improved with a strict face-wash ing reg imen and topical
3 antibiotics. Physica l examination revea ls numerous papules and pustu les with surrounding erythema on the
4 face, back, and chest. The doctor prescr ibes an oral antibiotic that is commonly used for treatment in similar
patients. The patient agrees to take the medication as prescribed and continues his routine of working long shifts
5
on the construction site. Four days after the prescribed medication is init iated, the patient develops severe
6 bl istering over his face, back of his neck, and the backs of his hands. He returns to the physician in a pan ic.
7
•8 What is the mechan ism of action of the most likely antibiotic prescribed in this case'
9
:
10 A. Binds the 50S subun it of the bacterial 70S rRNA
11
B. Binds to the 30S subun it of microsomal ribosomes
12
C. Binds to the enzyme DD-transpeptidase in the bacter ia l cell wa ll
13
14 D. I nteracts with the termina l D-Aia-D-Aia moieties of NAM/NAG-peptides
15 E. I nterferes with cross-l inking of bacter ial ce ll wa lls
• 16

6
lock
s
Suspend
0
End Block
Item: 16 of 16 ~. I • M k <:] t> al ~· ~
QIO: 2476 .l. ar Previous Next Lab 'lifllues Notes Calculator

2
The correct answer is B. 51°/o chose this.
3
Tetracycl ine binds to the 305 r ibosomal subunit and is associated with photosensitivity . Ultraviolet light is
4 absorbed by the drug, releasing energy and causing tissue damage on exposed areas, thereby explaining the
5 sunburn-like blistering the patient experienced because of working outside . Tetracycl ines are active against both
6 gram-positive and gram-negative bacteria but are now rarely used to treat bacteria l infections because of the
high preva lence of resistant organisms. Tetracyclines are now used primarily to treat acne vu lgaris, r ickettsial
7
inf ections, cholera, Lyme disease, and Chlamydia and Mycoplasma pneumoniae infections. This patient's job as
•8 a construction wor ker puts him at especially high risk for tetracycline-associated skin reaction .
9 The patient's history of a new drug with a known phototoxic side effect and the distribution of the eruption
10 being limited to sun-exposed areas suggest phototoxicity as the most likely diagnosis. However, blistering
11 eruptions can signif y more severe cond itions, such as Stevens-Johnson syndrome, and one shou ld be concerned
if the eruption are widely distributed, affect mucosal surfaces, and cover areas of skin not usua lly exposed to
12
the sun .
13
Lyme disease Mycoplasma pneumoniae Tetracycline Acne vulgaris Photosensitivity Cholera Gram-negative bacteria Phototoxicity Gram-positive bacteria
14
Tetracycline antibiotics Chlamydia (genus) Rickettsia Chlamydia infection 305 Mycoplasma Ultraviolet Bacteria Typhus Ribosome Mucous membrane
15
Adverse effect
16
A is not correct. 16% chose this.
Eryth romycin acts by binding the 505 subunit of the bacterial 705 r ibosom al RNA. Macrolide antibiotics are not
associated with the adverse effect of phototoxicity.
Erythromycin Macrolide Ribosomal RNA Antibiotics SOS Phototoxicity RNA Adverse effect Ribosome

C is not correct. 9°/o chose this.

6
lock
s
Suspend
0
End Block
Item: 16 of 16 ~. I • M k <:] t> al ~· ~
QIO: 2476 .l. ar Previous Next lab 'lifllues Notes Calculator

1
C is not correct. 9°/o chose this.
2
Penicil lin binds to the enzyme DD-t ranspeptidase, prevent ing t he linkage of peptidoglycans responsible fo r
3 building t he bacterial cell wa ll. Penici llins are not associated wit h sun - exposu re co mplications.
4 Penicillin Enzyme 00-transpeptidase Peptidoglycan Cell wall

5 Dis not correct. 11% c hose this.


6 Vanco mycin interact s wit h t he t ermina l D-Aia- D-Ala moieties of NAM/NAG-peptides. Wit h rapid intravenous
7 infusion, it can cause a severe anaphylact oid rash and red man synd rome . However, it is an inappropriate
•8 t reatment for t he clinica l picture in t his patient and is not associated with photosensit ivit y .
Vancomycin Photosensitivity Intravenous therapy Rash Moiety (chemistry)
9
10 E is not correct. 13% c hose this.
11 Monobact ams such as aztreonam are a t ype of ~ - lactam antibiotics, which wor k by interfering with t he cross-
linking of bact erial cell wa lls. Side effects of monobactams can include diarrhea, nausea, diffuse skin rash,
12
cand idiasis, and (rarely) serious adve rse event s such as angioedema and pseudomembranous col itis. They do
13 not, however, cause the adverse effect of phot osensit ivity .
14 Clostridium difficile colitis Aztreonam Monobactam Angioedema Candidiasis Diarrhea Photosensitivity Nausea Antibiotics Rash Adverse effect Colitis

15
16
Bottom Line:
Sunburn is a symptom of photosensit ivity, an adverse effect of tetracycline. Tetracycline binds to t he 30S
subunit of microso mal riboso mes .
Tetracycline Photosensitivity 30S Adverse effect Ribosome Symptom Sunburn

6
lock
s
Suspend
0
End Block
Item: 16 of 16 ~ 1 • M k -<:J 1>- Jil ~· !:';-~
QIO: 2476 ..L ar Pre v ious Next Lab fli!ltues Notes Calcula t o r

1 •
FA17 p 188.1
2 Tetracyclines Tetracycline, doxycycline, minocycline.
3 MECHANISM Bacteriostatic; bind to 30S and pre,ent attachment of aminoacyl-tRNA; limited C S penetration.
4 Doxycycline is fecally eliminated and can be used in patients with renal failure. Do not take
5 tetracyclines" ith milk (Ca 2+), antacids (Ca 2+ or vlg2+), or iron-containing preparations because
di,·alcnt cations inhibit drugs' absorption in the gut.
6
7
CliNICAL USE Borrelia burgdor{eri, M pneumoniae. Drugs' ability to <lccumulate intracellularly makes them ,·ery
effective against Rickettsia and Chlamydia. Also used to treat acne. Doxycycline effective against
·8
.\IIRS .
9
AOVERSEEFFECTS Gl distress, discoloration of teeth ;md inhibition of bone growth in children, photosensiti' it}.
10
Contraindicated in pregnancy.
11
MECHANISM OF RESISTANCE l uptake or t efAux out of bacterial cells b} plasmid-encoded transport pumps.
12
13
FA17 p 183.1
14
15
Antimicrobial therapy

16
GYRASE
FOLIC ACID SYNTHESIS
AND REDUCTION
(DNA methy~teonl Meltomdazole Fluoroquinolones
G _p_i"- ,-- - - - '
m Ciprofloxacin
Levofloxadn. etc
Sullonamides
Sulfamtthoxazole BACTERIALCEll ) Ouinolone
Sulfisoxazole PABA Nalidixic acid
SulfadiaZine

a
Lock
s
Suspend
8
End Bl ock
Item: 16 of 16 ~ 1 • M k -<:J 1>- Jil ~· !:';-~
QIO: 2476 ..L ar Pre v ious Next Lab fli!ltues Notes Calcula t o r

1 •
FA17 p 183.1
2
Antimicrobial therapy
3
4
GYRASE
FOUC ACID SYNTHESIS
5 AND REDUCTION
6 (DNA methylatiof'l)
~m__PI_n-,---__,) Ruoroquinolones
Ciprofloxacin
7 Sutfonamides levolloxacin. etc

·8 Sutfamelhoxazole BACTERIAL CELL Ouinolone


Sulfisoxazole Nalidixic aod
9 Sutfadwine

10
Tnmethopnm
11

12
13
14 50S SUBUNIT

15 Chloramphenicol
Clindamycin
16 CELL WAll SYNTHESIS
l inezotid
'EP 10 GLYCAN SYNTHESIS Macrolides
Azithromycin
Glycopeptides Clarithromycin
Vancomyc1n Erythromycin
8ac1trac~n Cell Wall Streptogramins
OUinupristin
Oalfopr1SIIn
PEPTIDOGLYCAN CROSS·UNKING
~< <J IRIINIT

a
Lock
s
Suspend
8
End Bl ock
Celt Wall Streptogramins
2 Ouinupristin
OalfopnstJn
3 P [) C YC•N CrOSS I~ Kl G
50S SUBUNIT
4 Penicillinase-sensitive peniciltins Antipseudomonal Carbapenems
Pell!CIIbn G. V Ticaroltin lm1penem Aminoglycosides Glycydines
5 Ampoctllln P1peracill1n Metopenem GentaiTIICin Ttgecyehne
Amoxic~hn Cephalosporins (1-V) Ertapenem Neomyon
6 Oonpenem Tetracyclines
Penicillinase-res~stant pelliCilUns lst-Cefazolln. etc Amikacin Tetracycbne
7 Oxacillin 2nd-Cefoxlbn. etc Monobactam.s Tobramycin Ooxycycbne
Nafalbn 3rd-Ceftriaxone. etc Aztreonam Streptomycin Minocycbne
·8
9
10
L D•cloxac•lhn 4th-Cefepime
Sth-Ceftarohne J D

11
FA17 p 146.3
12
Mycoplasma Classic cause of atypical ''walking" pneumoni<l o cell wall. 1 ol seen on Gram stain.
13
pneumoniae (insidious onset, headache, nonproductive Pleomorphic tl
14
cough, patchy or diffuse interstitial infiltrate). Bacterial membrane contains sterols for stabil ity.
15 X-ray looks worse than pcllienl. I Iigh liter of · 'lycoplasmal pneumonia is more common in
16 cold agglutinins (lgM), which can agglutinate patients < 30 years old.
or lyse RBCs. Grown on Eaton agar. Frequent outbreaks in military recruits and
Treatment: macrolidcs, doxycycline, or pnsons.
Auoroquinolone (penicillin ineffective since Mycoplasma gets cold without a coat (cell wall).
Mycoplasma ha\'e no cell wall).

a
Lock
s
Suspend
8
End Block
Item: 8 of 16 ~ 1 • M k -<:J 1>- Jil ~· !:';-~
QIO: 1922 ..L ar Pre v ious Next Labfli!llues Notes Calcula t o r

1 •
A 60-year-old woman with a history of peptic ulcer disease compla ins of intermittent joint pain and morning
2
stiffness for the past couple of years. Her hands have been affected the most, and she has recently had
3 difficulty gardening because of the pa in. There is no history of coronary or cerebrovascu lar disease. On
4 physical examination, the joints are tender to palpation, warm, and swollen. An X-ray of her hands is shown. An
anti-inflammatory med ication is prescribed.
5
6
7
·8
9
10
11

12
13
14
15
16

Wh;:tt is thP mPrh;:tni<:;m of ;:tl.tion of thP. rln1n most likPiv nrPsrrihPrl?

a
Lock
s
Suspend
8
End Bl ock
Image courtesy of William Scott, MD
11
12
What is the mechan ism of action of the drug most likely prescribed?
13
:
14
A. Decreased leukotriene production
15
B. Decreased prostag landin production
16
C. Decreased thromboxane production
D. Increased arachidon ic acid level
E. Increased prostacyclin level

a
Lock
s
Suspend
8
End Block
Item: 8 of 16 ~ 1 • M k -<:J 1>- Jil ~· !:';-~
QIO: 1922 ..L ar Previous Next Labfli!llues Notes Calculator

1 •

2
The correct answer is B. 68°/o chose this.
3
4 MEMBRANE
PHOSPHOLI'IDS
5
6
7
~
-

~
Tnamcuob If

UfDOPEROXIOE STtfTHESIS
I
8 Ar~aad Nf-•11 ~ (cy<l~

f t-
11<8

9 :....__ _---,
COX·2010LY rC:::OX:::.·l::..:C:::OX:::.·2

p"\. COX·Z ~ ~litr...,siblel


~

----
10 UUKOTRIEHE
Other NSAIDs!rovet>i>le)
5-Lipoxygenase
RECEPTOR
~
-
-=- Oiclofenac Ketorol.lc
11 ANTAGONISTS Cox 1 • -
lbUJ:(Offfl Naproxen
lndomeihiKin

12 Montelukdsl
5·HPETE Cyclic\ ndopcro•ides \
Zaftrluk.lst
13
1 \ 1
I
Leukotrienes Ptostacyclin Prostaglandms
~
Thromboxane
14
PGE 1 PGE2 PGF 20 TXA2
15 r bfonchial tone l nculrophll l platotot I vascular t uterine t vtctinc Tplatelet
chemotaxis aggregation tone tone tone aggregation
l vascular tone T vasculat tone
16
I [jlop•ostenot ) (irp•o~odiJ I Dinop<ostone ) I Carbop1ost )
The patient's symptoms, physica l exam, and X-ray findings are consistent with rheumatoid arthritis (RA). The
X-ray in the vignette shows erosions and narrowing of the joint spaces, as wel l as ulnar deviation of the f ingers
at the metacarpophalangeal joints. RA is an inflammatory condition that predominantly affects the metacarpal
joints, wrists, and knees.
~· . .' .. ,. .. ,. . ' • I .. r I • I

a
Lock
s
Suspend
8
End Block
Item: 8 of 16 ~. , . M k <:] t> al ~· ~
QIO: 1922 .l. ar Previous Next Lab 'lifllues Notes Calculator

1
The patient's symptoms, physical exam, and X-ray findings are consistent with rheumatoid arthritis (RA) . The
2 X-ray in the vignette shows erosions and narrowing of the joint spaces, as well as ulnar deviation of the fingers
3 at the metacarpophalangeal joints. RA is an inflammatory condition that predominantly affects the metacarpal
4 joints, wrists, and knees.
5 The current standard of care is to start the patient on disease-modifying antirheumatic drugs, of which
6
methotrexate is the drug of choice. However, it may take 3-6 weeks for methotrexate to take effect. This
patient needs immediate pain control, and thus the medication she is most likely prescribed is a nonsteroidal
7
anti-inflammatory drug (NSAID). Now, you have to figure in her existing peptic ulcer disease; which NSAID is
8 easiest on the stomach?
9 You shou ld be thinking celecoxib, a cyclooxygenase (COX) - 2 inhibitor. COX inh ibitors are anti-inflammatory
10 agents that block the conversion of arachidon ic acid to endoperoxides, wh ich are precursors to prostaglandins,
11
prostacyclins, and thromboxanes (see drawing). There are two types of COX enzymes . COX-1 is the dominant
isoform in gastric mucosal cel ls and is the major source of cytoprotective prostaglandin formation . I nhibition of
12
COX-1 leads to gastric damage. Therefore, since selective COX-2 inh ibitors, such as celecoxib, do not block
13 COX-1, they are associated with a low risk of GI toxicity.
14 Nonetheless, selective COX- 2 inhibitors are associated with increased risk of thrombosis and thrombotic
15 card iovascular events due to unbalanced thromboxane production in platelets. This is because selective COX-2
16 inhibitors do not block thromboxane production in platelets, as platelets only express COX- 1. Th is patient has no
card iac history and is at low risk of cardiovascular disease, so celecoxib would be a good first-line therapy for
her.
As for the other answer choices, only COX- 1 inhibitors have an effect on thromboxane production . COX-2
decreases (not increases) prostacyclin production. COX-2 acts upstream of arachidonic acid in the
endoperoxidase pathway, and does not affect leukotrienes, wh ich are produced via the lipoxygenase pathway .
Infliximab Etanercept Methotrexate Rheumatoid arthritis Arachidonic acid Cardiovascular disease Cyclooxygenase Prostaglandin Inflammation COX-2 inhibitor •

6
lock
s
Suspend
0
End Block
Item: 8 of 16 ~. , . M k <:] t> al ~· ~
QIO: 1922 .l. ar Previous Next Lab 'lifllues Notes Calculator

1 decreases (not increases) prostacyclin production. COX-2 acts upstream of arachidonic acid in the
2
endoperoxidase pathway, and does not affect leukotrienes, wh ich are produced via the lipoxygenase pathway .
Infliximab Etanercept Methotrexate Rheumatoid arthritis Arachidonic acid Cardiovascular disease Cyclooxygenase Prostaglandin Inflammation COX-2 inhibitor
3
4 Platelet Disease-modifying antirheumatic drug PTGSl Arthritis Thrombosis Peptic ulcer Thromboxane Metacarpophalangeal joint X-ray Circulatory system

5 Prostaglandin-endoperoxide synthase 2

6 A is not correct. 11% chose this.


7 Arachidon ic acid is converted to leukotrienes via the lipoxygenase pathway. This step is not directly affected by
8 the drug th is patient was most likely prescribed .
Arachidonic acid Leukotriene Prostaglandin lipoxygenase Complement system
9
10 C is not correct. 11% chose this.
11 Se lective cyclooxygenase inh ibitors are anti-inflammatory agents that block the conversion of arachidon ic acid
12
to endoperoxides, which are precursors to prostagland ins, prostacycl ins, and thromboxanes. There are two
types; one of them does block thromboxane production in platelets, but the other one does not. The type most
13 likely prescribed to th is patient does not affect thromboxane levels.
14 Arachidonic acid Cytochrome P450 Substrate (chemistry) Metabolism Prostaglandin-endoperoxide synthase 2 Drug metabolism Cytochrome

15
D is not correct. 6°/o chose this.
16 The anti-inflammatory drug this patient was most likely prescribed acts downstream of arachidonic acid in the
endoperoxide synthesis pathway. Thus it wou ld not be expected to dramatica lly increase arachidonic acid levels.
Arachidonic acid COX-2 inhibitor Peroxide Prostaglandin H2 Prostaglandin-endoperoxide synthase 2

E is not correct. 4°/o chose this.


The anti-inflammatory drug this patient was most likely prescribed does not increase prostacyclin leve ls.
Prostacyclin PTGSl COX-2 inhibitor Cyclooxygenase Prostaglandin-endoperoxide synthase 2

6
lock
s
Suspend
0
End Block
Item: 8 of 16 ~. , . M k <:] t> al ~· ~
QIO: 1922 .l. ar Previous Next Lab 'lifllues Notes Calculator

1
C is not correct. 11% chose this.
2
Se lective cyclooxygenase inh ibitors are anti-inflammatory agents that block the conversion of arachidon ic acid
3
to endoperoxides, which are precursors to prostagland ins, prostacycl ins, and thromboxanes. There are two
4 types; one of them does block th romboxane production in platelets, but the other one does not. The type most
5 likely prescribed to th is patient does not aff ect thromboxane levels.
Arachidonic acid Cytochrome P4SO Substrate (chemistry) Metabolism Prostaglandin-endoperoxide synthase 2 Drug metabolism Cytochrome
6
7 D is not correct. 6°/o chose this.
8 The anti-inflammatory dr ug this patient was most likely prescribed acts downstream of arachidonic acid in the
9
endoperoxide synthesis pathway. Thus it wou ld not be expected to dramatica lly increase arachidonic acid levels.
Arachidonic acid COX-2 inhibitor Peroxide Prostaglandin H2 Prostaglandin-endoperoxide synthase 2
10
11
E is not correct. 4°/o chose this.
12
The anti-inflammatory dr ug this patient was most likely prescribed does not increase prostacyclin leve ls.
Prostacyclin PTGSl COX-2 inhibitor Cyclooxygenase Prostaglandin-endoperoxide synthase 2
13
14
15 Bottom Line:
16 Se lective COX-2 inhibitors, such as celecoxib, are a type of anti-inflammatory medication that blocks the
conversion of arach idonic acid to endoperoxides, leading to decreased levels of prostag land ins and
prostacyclins. They are the anti-inflammatory agents of choice for patients at high risk of GI disturbances and
low risk of cardiovascula r disease.
Celecoxib Arachidonic acid Prostaglandin Anti-inflammatory COX-2 inhibitor Nonsteroidal anti-inflammatory drug Thrombosis
Prostaglandin-endoperoxide synthase 2 Prostacyclin Pharmaceutical drug Enzyme inhibitor

6
lock
s
Suspend
0
End Block
Item: 8 of 16 ~ 1 • M k -<:J 1>- Jil ~· !:';-~
QIO: 1922 ..L ar Pre v ious Next Labfli!llues Notes Calcula t o r

1 •

2 FA17 p 456.2

3 Celecoxib
4 MECHANISM Re,·ersibly and sclccti,·ely inhibits the cyclooxygenase (COX) isoform 2 ("Sclccoxib"), which is
5 found in inflammatory cells and vascular endothelium and mediates inflammation and pain;
spares COX-1, which helps maintain gastric mucosa. Thus, does not ha,·e the corrosive effects
6
of other ' SAIDs on the Cllining. Spares platelet function as TXA 2 production is dependent on
7 COX-I.
8
CLINICAL USE Rheumatoid arthritis, osteoarthritis.
9
ADVERSE EFFECTS f risk of thrombosis. Sulfa allerg).
10
11
FA17 p 456.3
12
Nonsteroidal Ibuprofen, naproxcn, indomethacin, kctorolac, diclofcnac, meloxicam, piroxicam.
13 anti-inflammatory
14 drugs
15 MECHANISM Reversibly inhibit cyclooxygcnase (both COX-I and COX-2). Block prostaglandin synthesis.
16 CLINICAL USE Antipyretic, analgesic, anti-inAammator)'. Indomethacin is used to close a PDA.
ADVERSE EFFECTS Interstitial nephritis, gastric ulcer (prostagland ins protect gastric mucosa), renal ischemia
(prostaglandins vasodilate afferent arteriole), aplastic anemia.

FA17p 439.1
Osteoarthritis and rheumatoid arthritis

a
Lock
s
Suspend
8
End Bl ock
Item: 8 of 16 ~ 1 • M k -<:J 1>- Jil ~· !:';-~
QIO: 1922 ..L ar Pre v ious Next Labfli!llues Notes Calcula t o r

1 •

2 FA17 p 439.1

3
Osteoarthritis and rheumatoid arthritis
4 Osteoarthritis Rheumatoid arthritis
5 PATHOGENESIS \ lechanical- wear and tear destroys articular Autoimmune-inflammation induces formation
cartilage (degenerati\ c joint disorder) of pannus (proliferati,·e granulation tissue ),
6
-+ inflammation with inadequate repair. which erodes articular cartilage and bone.
7
Chondroc}1es mediate degradation and
8 inadequate repair.
9 PREDISPOSINGFACTORS Age, female, obesity, joint trauma. Female, HLA-DR4, smoking, silica exposure.
10 ® rheumatoid factor (lgM antibody that
11 targets lgG Fe region; in 80%), anti-cyclic
12 citrullinated peptide antibody (more specific).
13 PRESENTATION Pain in weight-bearing joints after usc (cg, Pain, swelling, and morning stiffness lasting
14
at the end of the day), improving with rest. > I hour, improving with use. Symmetric
Asymmetric joint involvement. Knee carti lage joint involvement. Systemic symptoms
15
loss begins medially ("bowlegged"). o (rever, fa tigue, weight loss). Extraarticular

I
16 systemic symptoms. manifestations common."
JOINTFINDINGS Osteophytes (bone spurs), joint space narrowing, Erosions, juxta-articular osteopenia, soft I issue
subchondral sclerosis and cysts. Synovial swelling, subchondral cysts, joint space
Auid non-inflammatory (\.V BC < 2000/mm 3). narrowing. Deformiti es: cervical subluxation,
llwolves DIP (Heberden nodes : ) and PIP ulnar fin ger deviation, swan neck rn.
(Bouchard nodes B ), and 1st C MC; not .MCP. boutonniere (). ln\'Ok es i\IC P, PIP, wrist; not
J) JP n r l <t r.\Jr. <::vnm•i" J Anirl i n A<~ rnrn<~ lnn•

a
Lock Suspend
s 8
End Bl ock
Item: 8 of 16 ~ 1 • M k -<:J 1>- Jil ~· !:';-~
QIO: 1922 ..L ar Pre v ious Next Labfli!llues Notes Calcula t o r

1 ! - • t

2
*Extraarticular manifestations include rheumatoid nodules (fibrinoid necrosis with palisad ing histiocytes) in subcutaneous
tissue and lung (+ pneumoconiosis - Caplan syndrome), interstitial lung disease, pleuritis, pericarditis, anemia of chronic
3
disease, neutropenia + splenomegaly (Felty syndrome), AA amyloidosis, Sjogren syndrome, scleritis, carpal tunnel syndrome.
4
Normal Osteoarthritis Normal Rheumatoid
5 arthritis
- - - - -ThiCkened
6 capsule
~~~
/
7 / Slight syi10Y1al
Joint capsule """'\. Joint capsule """'\. eros1on
and S)'IIOVIclt \, / _hypertrophy and S)'llOVIcll \,
8
llning - - -Osteophyte IIrung
9 _ _ - Increased
Synovial . . / - -- Ulcerated S)'IIOVIcll _ / S)'llOVIcll RUid
10 ~ cart1lage CaVIty /
cavity/ _..-- Pannus
Cartilage ~ ~ SclerOilc bone Cartilage. / forma~on
11
Jomt space
12 narrowing
Subchondral
13 bone cyst
14
15
16

I
a
Lock
s
Suspend
8
End Bl ock

Вам также может понравиться